You are on page 1of 101
No ‘This material is copyrighted. All rights reserved. Edward Goljan, M.D. 2002 HIGH YIELD NOTES PATHOLOGY & = —_USMLE pictures: sec handout describing high yield pictures in Robbin's Sixth edition = USMLE hematology pictures not in Robbins: * supravital stain showing reticul filaments, © coarse basophilic stippling in Pb poisoning, » x-ray with density in ict RNA poisoning, « ringed sideroblast, * acute progranulocytic leukemia lots of Auer rede aoa tig’, DIC, « gross of non-Hodgkin's lymphoma encircling trachea and arch vessels wenn et them, + micro of B cell follicular lymphorna, » multiple myeloma marrow with plaeme ale urine, + Reed-Sternberg cell, * apple green birefringence of amyloid, » back of hand of elderly patient with senile purpura, + arms of elderly patient with multiple bruises~ probably abuse midbrain with yellow discoloration of kemicterus, + fetus with hydrops fetal i 2 — USMLE cardiovascular pictures not in Robbins: » xanthelasma, » Achilles tendon xan pathognomonic of familial hypercholesterolemia, » turbid supranate (chylomicrons) and inffanate (VLDL), « stasis dermatitis of lower leg~ sign of decp venous thrombosis, « Osler-Weber-Read discase— telangiectasia on fingers and mucous membranes, + spider telangiectasia, « lymphedema post mastectomy, * Kaposi's sarcoma, « micro of small vessel vasculitis with fibrinoid necrosis, « thromboangiitis obliterans, * patient with Henoch-Schénlein purpura, * gross of renovascular hypertension due to atherosclerosis, * capillary hemangioma on the face of a newbom, « Sturge Weber~ port wine nevus in trigeminal nerve distribution, « gross and micro of pulmonary edema, « coronary artery+ * identify platelet thrombus, + fibrofatty plaque, + dystrophic caleification, © atherosclerosis (empty slits), + section through ascending aorta~ what part dilated with postductal coarctation? proximal aorta, « rupture of posteromedial papillary muscle in an AMI, * rupture of anterior wall of LV in an AML, + ruptured interventricular septum in patient with an acute myocardial infarction rupture occurs in site of coagulation necrosis and leads to right heart failure, © mural thrombus in an AMI, « fibrinous pericarditis, « old AMI with scar tissue on anterior wall- do not confuse with a pale infarct, « valve vegetations- * MV vegetations in rheumatic fever: along the lines of closure of the valve, * infective endocarditis: see destruction of valve, + marantic vegetations on MV: * similar in appearance to rheumatic fever, + Hx of colon cancer or ulcerative colitis, + Libman Sacks endocarditis: occur on MV in patients with SLE, + gross of mitral valve « prolapse, * micro of myocardium with lymphocytic infiltrate~ coxsackie myocarditis © — USMLE respiratory pictures not in Robbins: « x-ray with spontaneous pneumothorax (tracheal shift to side of the pneumothorax) or tension pneumothorax (tracheal shift to contralateral side), « gram stain of Streptococcus pneumoniae, * x-ray or gross of lobar pneumonia, * gram stain of filamentous gram positive bacteria— Actinomyces or Nocardia (partially acid fast), « micro of macrophage with Histoplasma yeasts, « gross of cavitary apical lesion of TB~ possible acid-fast stain of organisms, + micro of a macrophage with acid-fast bacteria— TB, * micro of broad based bud of Blastomyces, « micro of wide-angled non-septate hyphae of Mucor, « gross of lung abscess, + micro of non-caseating granuloma in sarcoidosis, « gross of Hamman-Rich honeycomb lung, * x- tay of classic COPD, + gross of main-stem primary lung cancer 7 2 USMLE gastrointestinal pictures not in Robbins: + patient with Herpes simplex, + exudative tonsillitis, hairy leukoplakia, « thrush with or without gram stain of pseudohyphae and yeasts, « gross of two tongues with leukoplakia— one tongue is squamous dysplasia and the other is invasive cancer (look for asymmetry of tongue), « lower lip with squamous cancer, + Peutz-Jeghers lips, « patient with mixed tumor of parotid, « gross or barium study of Zenker's diverticulum, * gross of esophageal cancer~ think squamous if mid-esophagus or adenocarcinoma if distal esophagus, * X° ray of classic small bowel obstruction with air-fluid levels and step-ladder effect, + gross of intussusception in a child, * micro of absent ganglion cells in rectal biopsy in Hirschsprung’s, * Note: This material is copyrighted. All rights reserved. Edward Goljan, M.D. 2002 i o jodysplasia in cecum, * gross of carcinoid tumor (yellow c. ! sen pte ce a ott cern tobiliary/pamcreas pictures not in Robbins: * eyes with jaundice. « ee tal cee tan eee ey eet eae | phagocytosis,» gross of liver echinococcosis, « gross or micro of obstructive jaundice Hees green, look for stone in common bile duct, » Kayser-Fleische ring in Wilson's disease ven’ | total copper is low, ceruloplasmin is low, and free copper is high, » patient with Dupmreen contracture, + xray showing calcifications in the pancreas in chronic pancreatitis, © meee pancreatic cancer of head of pancreas USMLE genitourinary pictures not in Robbins: + casts in urine~ hyaline (fuzzy and round borders) vs waxy (sharp borders), RBC cast, renal tubular cast, fatty cast with Maltese rose cclls in urine~ WBCs, RBCs, + crystals in urine uric acid, calcium oxalate, cystine crystals, « horseshoe Kidney, * gross or x-ray of duplicated ureter, + gross of bilateral polyeysicladney discasc in a child, * gross of adult polycystic disease with or without brain with thes © subarachnoid bleed (ruptured berry aneurysm) or intracerebral bleed (hypertensive bleed), « linear immunofluorescence- think Goodpasture's syndrome, * lumpy bumpy IF— think ype I immunocomplex glomenalonephritis, « EM pictures fusion of podocytes in lipoid nephrosis, subepithelial deposits (post-streptococcal, membranous), subendothelial (type IV. SLE, membranoproliferative), * micro of nodular glomerulosclerosis in DM look for hyaline ancriolosclerosis of afferenvefferent and pink balls in mesangium, "Christmas ball” disease, micro of ischemic ATN- look for coagulation necrosis of tubular cells, « picture of kidney and show where Hg or Pb damages— proximal tubule, « gross of staghom calculus, « gross or micro or renal infarction— look for pale, sunken areas on the cortex and coagulation necrosis, & gross of benign nephrosclerosis— granular appearing cortical surface, * gross of malignant hyperteasion— look for petechial lesions on surface of the kidney, * micro of foreign body giant cell reaction in tubules BJ protein reaction in multiple myeloma, « gross of prostate hyperplasia— bulging nodular surface with occlusion of the urethra, * patient with Klinefelter’s— look for signs of feminization, « patient with testicular feminization looks like a woman but they will give history of blind vaginal pouch, * gross of renal adenocarcinoma~ yellow mass with hemorthage, yellow mass extending up the inferior vena cava USMLE gynecology pictures not in Robbins; * Herpes genitalis with or without a Tzanck prep showing multinucleated squamous cells with intranuclear inclusions, « condyloma acuminata— looks like a fern, HPV 6/11, ¢ Chlamydia- cell with vacuole in the cytoplasm containing an elementary body, # cervix or penis with thick exudate of GC with or without gram stain showing gram negative diplococei in polys, « clue cell of Gardnerella vaginalis— squamous cell covered by bacteria, « primary syphilitic chancre on penis or labia~ + painless, + if painful look for Hemophilus ducreyi, « picture of lymphogranuloma venercum— C. trachomatis subtype, papules with draining sinus from nodes, lymphedema, * Candida— cottage cheese discharge with or without gram stain of Pscudohyphae and yeasts, * squamous cell carcinoma of vulva— large ulcer on labia, * gross of cervical cancer, « micro of squamous dysplasia, « woman with hirsutism- * increased testosterone indicates ovarian origin, + increased DHEA-sulfate indicates adrenal cortex origin, * ovary with blood in cyst— endometriosis, * gross of ruptured tube (small embryo) from ectopic pregnancy, © ‘goss of follicular cyst, * gross or micro of serous cystadenocarcinoma of ovary- look for Psammoma bodies, * micro of signet ring cells with mucin in ovaries~ Krukenberg tumor from Stomach metastasis, © gross of abruptio placenta, * micro of choriocarcinoma- look for multinucleated cells representing syncytiotrophoblast and clear cells representing cytotrophoblast, « ‘&0ss of fibrocystic change, * patient with Paget's disease of nipple, « patient with inflammatory carcinoma of the breast with peau du orange appearance Note: This material is copyrighted. All rights reserved. Edward Goljan, M.D. 2002 USMLE endocrine pictures not in Robbins: » patient with cicst visual field defect, which is bitemporal hemianopsia, = inte ears cyst ae age thyroglssal duct eysh «think branchial clet cyst if iis in the anterolateral nee eR * ticro of papillary thyroid eancer~ look for psamsmoma bodies, « patent sith Gre oe ise act cxophhalmos,» patient with Hashimoto's thyoidits~ lock fr penrbital por me thumb adducted into palm~ tetany, * micro of adrenal cortex/medulle ee, synthesizes mineralocorticoids, * ona fasciculata synthesizes, corti ‘synthesizes sex hormones, * adrenal medulla synthesizes catecholamines, * patient with Cushin; syndrome look for purple stra, « brown colored tumor in adrenal medulla pres id Patient with buccal mucosa hyperpigmentation— Addison's with increase in ACTH, © child with arebiguous genitalia~ 21 or 11 hytroxylase deficiency, foot with dry gangrene in'¢ mance coagulation necrosis, » retina with microaneurysms in a diabetic duc to somone damage of Pericytes, « well circumscribed tumor in adrenal cortex- could be Cushing's, Cons syndneen, oe non-functioning adenoma depending on the question = * USMLE musculoskeletal pictures not in Robbins: + synovial fuid erystls~ + yellow and parallel to slow ray is MSU in gout, * blue and parallel is calcium pyrophosphate in pseudogout, « chunky crystal in calcium pyrophosphate, « patient with osteoarthritis of hands Heberden's nodes 'n DIP joint « patient with zheumatotd arthritis in the hands, « patient with inflamed big toe_ acwee Sout, ¢ patient with tophus with or without microscopic showing multinucleated giant celle and Polarizable material, + patient with ankylosing spondylitis, « patient with Reiter's syndrome who has conjunctivitis with or without history of Achille’ tendon periosttis, « gross of osteomyelitis with or without picture of sickle cells + Salmonella osteomyelitis, * if no HX of sickle cell, Staphylococcus aureus is the answer, # young patient with inflamed knee with or without pustules on wrists or feet~ disseminated gonococcemia, » patient with Lyme's disease and rash of erythema chronicum migrans, » patient with blue sclera of osteogenesis imperfecta « defect in synthesis of ‘ype I collagen, * blue color due to loss of collagen in sclera and visualization of underlying choroidal veins, + gross of bone with osteoporosis, « MRI of the knee- know how to identify anterior and posterior cruciate ligaments, © gross of osteochondroma cartilage overlying bone Protuberance, * micro of muscle atrophy, * patient with Duchenne's muscular dystrophy, « patient, with myotonic dystrophy— * look for drooping mouth and frontal baldness, * trinucleotide repeat disorder, * patient with myasthenia gravis— look for drooping eyelids, « gross of lipoma, LE cell- neutrophil with phagocytosed red staining altered DNA, * seram ANA with rim pattem— SLE Patient with anti140 mg/dL to >126 me/dL) ‘on sensitivity, specificity, PV", and PV: ‘Note: This material is copyrighted. AIl rights reserved, Edward Goljan, M.D. 2002 1, increases sensitivity and negative predictive value (PV) — « dro the reference interval to a lower vale means that more people wine vee Upper limit of likely tobe TNs (not have DM) rather than FNS, » sensitivity and PY ene fet Ul ae the upper limit of a test reference interval is lowered YS increase when 2. decreases specificity and positive predictive value (PV) « fewer pe DM, a test result 7126 mg/AL is more likely to be a FP than Te as schematic It, © summary Normal _Disease 0 4 0 Interval O- 4 Sensitivity 100% (no FNs) PV 100% Specificity decreases PV" decreases —_—_——_> Interval 0-10 Specificity 100% (no FPs) PV" 100% Sensitivity decreases PV decreases = Prevalence: 1. Prevalence (number of people with disease in the population studied) = Incidence (number of new cases over a period of time) x Duration of the disease- P =I xD, «as duration (D) decreases, prevalence (P) decreases, « as D increases, P increases, » incidence (1) is. 2 constant in this relationship 2. prevalence calculation TP + FN (all people with disease)/ TP + FN + TN + FP (all people with and without disease} 3. example— if treatment for leukemia lengthens the survival period but does not Iead to its ‘cure, prevalence (P) of leukemia increases owing to the increase in duration (D)- no effect on incidence (aumber of new cases of leukemia) @ Example of a calculation for Sensitivity, specificity, PV’, PV, prevalence: Disease No Disease Rositive test (TP) 60 (FP) 40 ‘Negative test (EN) 20 (TN) 80 ‘Sensitivity of the test: TP / TP + EN = 60/80 = 75% Specificity of the test: TN / TN + FP = 80 / 120 = 66% PV": TN/ TN + EN = 80/ 100 = 80% (80% chance it is a TN and a 20% chance it is FN) PV: TP/TP + FP =60/ 100 = 60% (60% chance itis a TP and 40% chance it is a FP) Prevalence: TP + FN/TP + FN + EN + EP = 80 /200= 40% & Normal changes in pregnancy: |. greater increase in plasma volume than RBC mass + decreases hemoglobin (Hb) and hematocrit (Het) dilutional effect, « increases glomerular filtration rate (GFR) and creatinine clearance (CCr)- due to increased plasma volume, « decreases serum BUN/creatinine/uric acid- dilutional effect + increased renal clearance | Note: This material is copyrighted. All rights reserved. Edward Goljan, M.D. 2002 2. increased alkaline phosphatase~ placental origin respiratory alkalosis~ estrogen/progesterone increased clearance of CO; per breath 4. increased T, and cortisol- globulin, transcortin, respectively), free hormone. leven pid hyperthyroidisny’ hypercortisolism~ e.g., normal serum ‘* Main laboratory difference in adult male and female oy €.8., serum iron and ferritin, * lower Hgb concentration iy = Children: 1. imereased serum alkaline phosphatase (ALP) « 3-: release enzyme when stimulated by vitamin D 3.__ slight decrease in hemoglobin concentration whex % Newborn: high hemoglobin (Hgb) due to increase in Hgbr-- Left shifts oxygen dissociation curve (ODC)~ causes tissue hypoxia 2. stimulus for erythropoietin (EPO) release» i pprnsresses RBC production wich subsequent increase in Hgb concentration = HgbF: eat shifts oxygen dissociation curve (ODC) 2 Protects newborns with sickle cell disease « most of the RBCs at birth contain HOF inhibits sickling, « less HgbS- + concentration not high enough for sickling, + HgbS must be om in RBC for spontaneous sickling, « dactylitis (bone infarctions Of digits) begins in 6-9 ni 3 aly eabF synthesis is increased with hydroxyurea used to reduce sickle cell crises * Analytes increased with hetnolyzed blood sample secondary to veuipunctare. TDE« isoenzyme fraction is primarily increased and is greater than LDH, isoenzyme fraction (LDHy/LDH, flip), « false positive diagnosis of acute myocardial infarc” LDH, isoenzyme is also in cardiac muscle 2. potassium pseudohypcrkalemia~ false increase, « K*is the major intracellular cation, « ECG will not show a peaked T wave © Lipids and fasting: 1. triglyceride (1G) component coming from chylomicrons falsely increases serum TG levels chylomicrons contain diet-derived TG, hence the patient must be fasting wo eliminate this variable . 2% fasting or lack of fasting does not affect cholesterol (CH) and high-density lipoprotein CADE) concentration— » normally, CH is <3% of the chylomicron fraction, « fasting is unnecessary for an accurate CH or HDL 3. fasting is necessary for an accurate calculated low-density lipoprotein (LDL)- + LDL = CH - HDL - TG/5, if TG is falsely increased by chylomicrons from the dict, it will falsely lower the calculated LDL , . ‘Drugs eahancing the cytochrome system im the liver smooth endoplasmic reticulum (SER): 1. drugs alcohol, barbiturates, rifampin, phenytoin - 2 eftecton SER- + SER bye agi, reseed synthesis of y-glutamyltransferase (GGT)- enzyme is normally located in SER, * decreases drug levels owing to increased metabolism of the drug . . 3. elinieal scenario- woman on phenytoin who is also taking birth contro! pills gets pregnant: Phenytoin revved up the SER and increased metabolism of birth coatrl pill ‘© Drugs inhibiting cytochrome system in the liver: 1. drugs— Hy blockers, proton blockers Note: This 2 = Signi process material ls copyrighted. All rights reserved. Edward Goljan, MD. 2002 danger of drug toxicity ificance of erythrocyte sedimentation rate (ESR) in old 38°: probably indi , ICAteS & disease & Laboratory test alterations in alcoholics: 1 2 bor q Bee PS be hie fa enhancement of the liver cytochrome P-450 system. « ; — gintamylranferass (GGT) excellent enzyme marker fr rcaga liver disexee “S Of 1+ increased production of NADH in its metabolic breakdows causes biochem involving NADH to move in ies direction resulting in toe following ee hiram pyro ate © fasng eat oruvate is unavailable fr gloconespe so iy lyeeridemia~ 1,3 bisphosphoglycerate-> dih . bypereabie "aroxyacctone phosphate-> slyeert a, increase in ketoacid syuthesis— acety] CoA, the end luct of in the following reactions: acetyl CoA + acetyl Goa acne ne a ae acetoacetic acid increase in NADH converts it into 8 hydroxybutyric acid (BOHR) ae Acid syuthesis— due to the increase in acetyl-CoA, the initial substrate for hyperuricemia lactic acid/kctoacids compete with uric aid for ‘xeretion in the kidneys increased anion gap metabolic acidosis— lactate + B-OHB ratory test alterations in smokers: prsPiratory acldosis~ air gets in but cannot get out, so CO, is retained hypoxemia (low P20,) Increased carbon monoxide (CO) levels—-CO is present in cigarete smoke secondary polycythemia~ low PaO, stimulates eryehtopoietin wlan apsoine neutrophilic leukocytosis metabolites in smoke mebilize the neutrophil yee natng poo! in the cteulation by decreasing leukocyte adhesion fo eae vat cells Plasma/serum turbidity: due to an increase in triglyceride (TG)- turbidity does not occur with an increase in cholesterol (CH) in plasma 2 TG is carried by lipoproteins~ + chylomicrons 85%, « very low-density lipoprotein (VLDL)- 55% 3. TGis falsely increased after eating— due to diet-derived chylomicrons 4. chylomicrons form a supranate in plasma— contain very little protein: less dense than VLDL 5. VLDL forms an infranate (no supranate)- contains more protein than chylomicrons and does not float on the surface of plasma 6. _ imcreased turbidity interferes with measurement of enzymes and serum Na*~ falsely low enzyme values and sodium (pseudohyponatremia) Supranate Infranate (VLDL) (chylomicrons) 2. as ‘& Relation of serunt albumin concentration with Serum calcium concentration: 1. albumin binds 40% of total calcium in blood. « 13% of calcium is bound to other Substrates, « 47% calcium is free, ionized calcium metabolically active calcium i ‘ow serum albumin decreases calcium bound to albumin « hypocatcemia, « noteuny is Present, since the ionized levels are normal, « PTH is normal o o 94 owessan, MLD, 2002 Lyon's hypothesis: 1. one of the 2 X chromosomes in a female is chromosomes are matemal, « ~50% X chromosomes omes a Barr body- projection from the : eae from the buccal mucosa nucleus counted in s males have J Barr body and norm: Klinefelter syndrome (KXY. Toor Ene ae Barr bodies . female with Turner's syndrome (XO) has no Barr bodies : lolecular genetics: stic fibrosis— » deletion of 3 nucleotides coding for transmerabrane reguletor is defective and ie degaced bee ae aed CF after it leaves the Golgi appararus ing the cell membrane Tay Sachs— = 4 base insertion produces a frameshift mutation hexosaminidase 7 3. sickle cell disease/trait— point mutation involving thymidine . valine instead of glutamic acid in 6th postion of Pglobin chaig Pea? SERINE codes for 4. Betbalassemia major point mutation produces a stop codon leading to termination of DNA transcription of B-globin chain 5. trinucleotide repeat disorders— + progressively worse disease occurs in future generations (phenomenon is called anticipation), « constant repetition of 3 nucleotide bases (gs CAG, CAG, CAG etc.) « more trinucleotide repeats are added each generation leading to progressively worse disease and earlier manifestation of the disease, + examples Huntington's disease (AD), fragile X syndrome (SXR, female carriers with timeleoude repeats eventually become symptomatic), Friedreich's ataxia (AD), myotonic dystrophy (AD) Nondisjunction: + cause of the majority of chromosome number disorders (¢.g., tisomy 21, Tumer’s XO), * due to unequal separation of chromosomes in first phase of meiosis Mosaicism: # nondisjunction of chromosomes in mitotic division in the early embryonic period, « two chromosomally different cell lines are derived from a single fertilized egg, « most cases involve sex chromosomes—e.g., gonadal dysgenesis with XO/KX, XO/XY Translocation: * one part of 2 chromosome is transferred to a non-homologous or homologous chromosome, * called a balanced translocation if the translocated fragment is functional, Robertsonian translocation in Down syndrome: + type of balanced translocation with a reciprocal translocation between 2 acrocentric chromosomes (Ome), » usually chromosomes 21 and 14, * produces 1 long chromosome (14=21): extremely small translocated chromosome usually lost, # mother has the translocation: 45 chromosomes with 1 long #14+21, 1 normal #14, and 1 normal #21, + mother is normal, since both translocated fragments are functional, « Down child receives 1 normal #21 from uninvolved parent (father) and | 14*21 and 1 normal #21 from affected mother for a total of 46 chromosomes but all 3 #21s are functioning Ben “z = * codes for defective 21 2 4 14 2 2 “| TT It Parent: 45 chromosonies Down syndrome: 46 een em cat Microdeleton see a eras pom elcomosome can oly ieniied wh i . tion from 1 josome fue Microdeletion syndro: loss of a small portion 1 Oe ee high resolution techniques, + microdeletion on chromosot 5 Lsesaoien, syndrome~ * chromosome 15 deletion is of patemal_origin, obesity, ST ace retardation, + microdeletion on chromosome 15 may result in Angelman syndrome * < Note: This material Is copyrighted. AU rights reserved, Edward Goljan, M.D, 2002, 15 deletion is of matemalorigin. * “happy puppy” syndrome (chi cannot talk), « term applied to these syndromes in (child always happyfaughing but © Risk of recurrence of genctle disorders: Mendelian disorders have the recurrence: AR diseases have # 25% recurrence rate Sreatent risk of * Genetics disorders in African-Amerteans: « sickle cell rnitidixease— cell gene, © avp-thalassemia, * glucose G-phosphate dehydrogena hereditary persistence of UgbF? @ — Genetle disorders in Ashkenazt Jews: « factor XI deficiency, 10% prevalence of sickle se (GOPD) deficiency, « * Gaucher's disease, « Tay-Sachs discase © Genetic disorder ta Northern Europeans: cystic fibrosis~ MC genetic disease in patient's ability to. ronraduce owing to early death or problems with feriig netme ah the @ Genetic disorders in Mediterrancan peoples: + Ge ” Genetic aisorde peoples: * GORD deficiency,» sickle cell taivdisease,« pe Genetic disorder im Southeast Asians: o-thalasscmia MC genetic syndrome associated with advanced maternal age: trisomy 21 Down syndrome pathogenesis: trisomy 21- * 95% of all cases, origin for extra chromosome, Robertsonian translocation— + 4%, mother with 45 chromosomes- usually mother, 25-50 in 80% ‘* Down's clinteal findings: « epicanthal folds with upward slanting, simian paimar crease, « poor reflexes/nypotonicity, + cardiovascular * endocardial cushion defects (combined ASD and VSD), ing factor for survival in carly infancy and childhood, * GI- + duodenal atresia (polyhydramnios, vomits bile at birth, double bubble sign on x-ray), * Hirschsprung's disease , « hematologic~ increased incidence of leukemia, « CNS- * Alzheimer's disease, * chromosome 21 codes for B-amyloid proteins, which are converted into amyloid and are toxic to neurons, « universal by age 35 (any patient with Alzheimer’s discase under 40 is a patient with Down syndrome), reproductive + all males are sterile, + females have 50% chance of having a child ith Down’ © — Risk for future children with Down's: * 1-2% overall risk for trisomy 21, ¢ matemal age- women >35 yrs of age, + 5-15% risk for parent with a balanced translocation, © karyotype of affected child should always be determined to evaluate risk for siblings to have affected children * Trisomy 18 (Edward's syndrome): + scvere mental retardation, © clenched hands_with overlapping 2nd and Sth fingers, + rocker bottom feet © Trisomy 13 (Patau’s syndrome): * cleft lip/palate, + severe mental retardation, » polydactyly, * cystic kidneys = Mendelian disorders in descending order of frequency: * autosomal dominant (AD), * autosomal recessive (AR), * sex-linked recessive (SXR), * sex-linked dominant (SXD) = AD disorders: + only 1 abnormal allele is necessary to express the disease- + "dominant gene”, * €¢.g., aa (disease, usually lethal) or Aa (disease), © only on¢ parent has to have the gene to pass \ton to their children, * one affected parent + normal parent 50% normal children, 50% affected children * AR disorders: ¢ both abnormal alleles must be present (homozygous) to express the disease * 3% (disease), + Aa (heterozygote asymptomatic carrier), + both parents must have the abnormal allele= + 2 asymptomatic carriers (children- 25% normal AA, 50% Aa, 25% aa), * patient with the disease has children with an asymptomatic carrier (children 50% Aa, 50% aa) . 7 — SXR disorders: « males with abnormal allele express discase~ males are "homozygous", since he only have 1 X chromosome, © affected males transmit the disease to both daughters but none a their sons daughters are usually asymptomatic carriers, » female carrier transmits the disease 944 * 47 chromosomes, * maternal * 46 chromosomes in child, « + MC genetic cause of mental retardation 1Q Note: This material is copyrighted. All rights reserved. Edward Goljam, M.D. 2002 50% of the boys and 25% of daughters are asymptomatic 3 3 50% of males are affected, 25% of females are asymptomatic oe symptomatic- maternal and paternal X chromosome are abnormal s SXD disorders: males and heterozygous fernales both express the disease AD disorders: « associated with structural defects in proteins and receptors~ * enzyme deficiencies are uncommon, * AD disorders that are enzyme deficiencies acute interminent porphyna (deficiency of uroporphyrinogen synthase), * hereditary angioedema (C1 est taba deficiency), « late manifestations of disease~ c.g., Huntington's disease with chorea/dementia later in life, ¢ exhibit penewance- * person has the abnormal gene but never expresses the disease. however, the gene is transmitted to their children, * some AD disorders have 100% penetrance (€.g,, familial potyposis, adult polycystic kidney disease), while others have incomplete penetrance (cg., Marfan's), © exhibit variable expressivity affected people have different levels of severity of the disease, » mechanisms of AD disease without a family Hx: * MCC is incomplete penetrance, « new mutation, » examples of AD diseases (in order of decreasing frequency): + von Willebrand's disease + familial hypercholesterolemia, adult polycystic kidney disease, hypertrophic cardiomyopathy, + Huntington's disease, + neurofibromatosis, * congenital spherocytosis, * familial polyposis, * acute intermittent porphyria, + osteogenesis imperfecta, * Marfan syndrome = — Example of a pedigree with complete penetrance: 94 © — Example of a pedigree with incomplete penetrance: @ AR diseases: ¢ no evidence of penetrance, variable expressivity, late manifestations, « most (not all) AR discases are enzyme deficiencies~ inbom errors of metabolism- * acute intermiment porphyria and Cl esterase inhibitor deficiency are both autosomal dominant, * G6PD and Lesch- Nyhan are both SXR, « examples of those that are not enzyme deficiencies~ * cystic fibrosis, * sickle cell waiv/disease, * hemochromatosis, * Wilson's discase : Not o te: This material is copyrighted. Alll rights reserved. Edward Goljan, M.D, 2002 Example of an AR pedigree: [_} SXR disorders: examples (in order of decreasing frequency): © fragile X syndrome, « deficiency, * Duchenne’s muscular dystrophy, * hemophilia A/B, » sso cabin immunodeficiency, + Wiskott-Aldrich syndrome, * testicular feminization, » color bie s chronic granulomatous disease of childhood, «Bruton’s agammaglobulinemis ° Fragile X syndrome: » some geneticists say itis a sex-linked dominant disease, owing to the fact that female carriers may express the disease in future generations (concept of anticipation), = second MC genetic cause of mental retardation, « MC Mendelian disorder associated with mental retardation, * clinical- * mental retardation, * macroorchidism— at puberty, ¢ ~30% of female camiers are mentally retarded or have impaired leaming- due to anticipation and addition of Sinucleotide repeats with future generations, * abnormal fragile X chromosome- DNA analysis for carrier identification (identify CGG trinucleotide repeat is the best test to confirm) Lesch Nyhan syndrome: * SXR disease with a deficiency of HGPRT- no inhibition of PRPP in purine metabolism, « clinical- * hyperuricemia, * mental retardation, * self mutilation SXD disorders: « percentages of children with the abnormal allele are the same as those in SXR disorders: + dominant abnormal allele causes disease in both males and females, « affected woman transmits symptomatic disease to 50% of her daughters and 50% of her sons, + affected males ‘transmit symptomatic disease to all of their daughters and none of their sons, « examples: + familial hypophosphatemia (defect in the proximal reabsorption of phosphate and conversion of 25(OH)D, to 1,25 (OH)rD;), * Alport_syndrome (hereditary glomerulonephritis associated with nerve deafness) Multifactorial (polygenic) inberitance: © multiple small mutations plus the effect of environment- * should be suspected when there is an increased prevalence of disease among relatives of affected individuals, + parents and offspring have 50% of their genes in common, + enzymes involved in ndria— Calculation of the prevalence of 2 genetic disease given the carrier rate: , i (CF), with a carrier rate of CF is 1/25— * number of couples at risk is equal - Gene males x the carrier rate in females, or 1/25 x 1/25 = /625 couples are at risk, * risk of having a child with CF (AR disease) is 1/4, hence 1/625 x 1/4 = prevalence of ~1/2500 Simple way of calculating the carrier rate of a disease given the Prevalence of a genetic disease (Hardy Weinberg equation): + reflects the distribution of a mutant gene in the population, example using prevalence of CF of 1/2500-« number of couples at risk = 1/2500 + U4 = 625, + carrier rate of CF = 1/V625 = 1/25 ? Deformations: * anatomical defects resulting from mechanical factors (extrinsic forces) that usually occur in the last two trimesters after organs have developed, + e.g., oligohydramnios Producing facial and limb abnormalities (called Potter's facies) Malformation: * disturbance (c.g, drugs, infection) that occurs in the morphogenesis of an organ(s), + hypospadias faulty closure of urethral folds, + epispadias due to a defect in genital tubercle, « club foot, © ventricular septal defect Mechanismis of teratogens: « teratogens are most detrimental during the embryonic period~ first 9 weeks of life (4th-Sth week most sensitive for teratogens), * ¢.g., open neural defects occur when tube normally closes between the 23rd~28th day, * specific effects of some teratogens: * some interfere with formation of mitotic spindle, + interfere with production of ATP, some interfere with gene production (¢.g., isotretinoin effect on Hox/hedgehog genes, which are important in embryonic patterning) Teratogen cocaine: * maternal effects: + hypertension, + abruptio placenta, + newborn effects: * hyperactivity, * microcephaly (MC effect), * interruption of blood flow leading to infarction (CNS, bowel, missing digits) Maternal diabetes mellitas aud the teratogenic effects in newborns: © increased birthweight (macrosomia, large for gestational age): * hyperinsulinism in the ferus from poor maternal glycemic control increases muscle mass (insulin increases amino acid uptake in muscle), + hyperinsulinism increases fat deposition (insulin increases deposition of TG in adipose), * open neural tube defects, © cleft lip/palate, * respiratory distress syndrome: fetal hyperinsulinism in response to maternal hyperglycemia inhibits fetal surfactant production by type II pneumocytes . Teratogen diethylstilbestrol (DES): * mothers received DES to prevent threatened abortions, * DES interferes with the development of miillerian structures in female fetus causing abnormalities in the tubes, uterus, cervix, upper one-third of vagina, « female siblings: + vaginal adenosis is the MC abnormality and is the precursor of clear cell adenocarcinoma of the vagina/cervix, ¢ cervical 13 Note: This material is copyrighted. All rights reserved. Edward Goljan, M.D, 2002 incompetence~ increased incidence of spontaneous abortions, « ; with implantation, « fallopian tube abnormalities fertility problema nT Alities— problems = Fetal alcohol syndrome and teratogenic effects in newborns: « States: 2:1000 live births, + clinical + mental retardation (MC Sena a in United recardation, + maxillary hypoplasia, microcephaly, atrial septal defeets (Ieast o ine growth + hypoglycemia at birth -ommen finding), % Teratogen heroin: » srall for gestational age, » irritabilityyperactivty, excessive hunger, salivation, sweating, tremors, fist sucking, «temperature instability, « se 7 Teratogen isotretinoin: * used in treating cystic acne~ * must order a pregnancy corto Placing a woman on the drug, * patient must be on birth control pills while taking the aaice newbom effects (3 Cs): * craniofacial abnormalities (small ears), micrognathia, cleR paler,» cardiac defects, CNS malformations (hydrocephalus, microcephaly) palate), ‘7 Teratogen phenytoin: « nail hypoplasia, « CNS abnormalities, « cleft lip/palate, « ae ip/palate, * congenital heart 7 Maternal smoking: * vasoconstrictive effects of nicotine produce placental i = damage increases the risk for thrombosis in placental vessels low birth pues ee Mothers with SLE: SLE patients who have anti-Ro - i i have newboms with complete heart block {en SS-4) 1eG ansbodies in heir serum may * Teratogen thalidomide: + previously used in the United States to control nausea associated with pregnancy— currently used in Rx of leprosy, © limb abnormalities~ * amelia (absent limbs), + phocomelia (seai-like limbs) = — Teratogen valproate: open neural tube defects © Teratogen warfarin: « contraindicated in pregnancy: should use heparin, * CNS defects, « nasal hypoplasia Congenital infections: « TORCH syndrome: + Toxoplasmosis, + Other (HBV, AIDS, parvovinus, syphilis, etc.), + Rubella, + CMV (cytomegalovirus), + Herpes, « increase in IgM in cord blood, + vertical transmission (mother to fetus)~ * transplacental MC type, * blood contamination during delivery(c.g., HBV), * cervical infection (HSV 2, Chlamydia), + breast feeding (HIV, HBV, CMV) Congenital CMV: + MC_in-utero viral _infection: majority are asymptomatic, * primarily transplacental, « clinical~ + bilateral sensorineural hearing loss MC complication, + periventricular calcification, + hepatosplenomegaly, chorioretinitis (may lead to blindness), « urine culture is gold standard- urine cytology reveals large, basophilic intranuclear inclusions (“owl eyes”) in renal tubular cells, * Rx- * ganciclovir (begin with this), + foscamet (if the former is not working) ‘Congenital rubella: « primarily transplacental— highest incidence of congenital anomalies in first 8 weeks- virus interferes with protein synthesis and produces a vasculitis, » clinical- * sensorineural deafness MC complication, * cataracts, * patent ductus arteriosus Congenital toxoplasmosis: « primarily transplacental, + contracted by women after exposure to cat litter * pregnant women should avoid cleaning cat liter during pregnancy, * contracted also by handling or eating undercooked meat products, * greater risk of fetal infection later in pregnancy than earlier in pregnancy, © clinical- * chorioretinitis MC late complication (often leads to blindness), « calcifications in basal ganglia, « Sabin Feldman dye test (uses live organisms)}- "gold standard" test but is rarely performed Congenital Herpes type 2: + primarily contracted by passing through the birth canal in women actively shedding the virus- * women actively shedding the HSV-2 virus are delivered by C- section, * greater chance of fetal infection with primary rather than recurrent Herpes, « clinical~ local or systemic disease (encephalitis, skin infections), « Rx: acyclovir * high pitched cry with Congenital syphilis: * primarily transplacental— fe Brcenaiey~ anatomical borers prevent access tothe Teal caine ee SOM of (first 2 yrs): * hepatomesaly MC sign, * osteochondritis (inflammation often, ie lesions, + pneumonia alba (lobar pneumonia), persistent rhinitis (muffle), lane Se (2 yrs: + frontal bossine_is MC sign, + saber shins, + rhagadet (cenieal nen ils Murhinsons id eth (necked upper ental incisor called Hutchinson's tech nd nels molars called mulberry molars), + interstitial keratitis (blindness), * sence Rx: penicillin > ™ 1 loss, « Congenital Varicella-zoster virus: + chorioretinitis— cortical atrophy in the brain, « vesicular skin lesions Age dependent disorders (Inevitable with age): + increased body fat: decreases insulin receptors leading to glucose intolerance, * res} howe ore jiratory— * ob: ‘ pulmonary function tests (so-called “senile apliiand jgheneers alti Cee expiration), * decreased forced expiratory volume in 1 second (FEVise), * decreased forced vital capacity (FVC), * increased total lung capacity (TLC), + increased residual volume VY), * low normal PaO;, « cardiovascular * loss of elasticity in aorta, + decreased cardiae output ard heart ate in response to sttess, * at rest, the cardiac output is unchanged, * musculoskeletal Osteoarthritis in weight bearing joints, *CNS— + cercbral atrophy with mild forgetfulness, = impaired sleep pattems (insomnia, early wakening), Parkinsonian-like gait, « sensory changes- * cataracts, * arcus senilis (ring of cholesterol around the comea), + presbyopia (inability to focus on near objects), * presbycusis (sensorineural hearing loss particularly at high fiequency), « otosclerosis (fusicz. of the ear ossicles producing conductive hearing loss), + immune system « decreased T cell function (often anergic to common skin antigens, positive PPD non-reactive or absent), + loss of isohemagglutinins to blood antigens (may not develop a hemolytic transfusion reaction if blood is mismatched), « skin changes + loss of skin elasticity (increased cross-bridging of collagen), + senile purpura over the dorsum of the hands (common USMLE picture) and lower legs where bumping into objects is most likely to occur, + decreased sweating (eccrine glands fibrosed, danger of heat stroke), # GE tract- + decreased gastric acidity, + decreased colonic motility (constipation which predisposes to diverticulosis), + decreased activity of the hepatic cytochrome P450. system(danger of drug toxicities), + male reproductive + prostate hyperplasia (increased urine residual volume with subsequent increase in urinary tract infections), * prostate cancer (only cancer that is age dependent), * decreased testosterone,+ female reproductive— » breast and vulvar atrophy due to decreased estrogen, * increased gonadotropins, « endocrine system— increased glucose intolerance (due to increase in body fat and subsequent reduction in insulin receptor synthesis), © renal changes~ decreased GER with reduction in the creatinine clearance (risk of drug toxicity due to slow clearance of drugs), Age-related changes (increased incidence in elderly but not inevitable): + cardiovascular- atherosclerosis with increased incidence of coronary artery disease, peripheral vascular disease, aad strokes, * temporal arteritis, * aortic stenosis (MC valvular abnormality in the elderly), * systolic hypertension due to loss of aortic elasticity, « musculoskeletal system— « osteoporosis (particularly the vertebral column in females and femoral head in males), * Paget’s disease of bone, * respiratory system- pneumonia usually Streptococcus pneumoniae (underscores importance of Paeumovax vaccination in elderly), « CNS— * Alzheimer’s disease, * Parkinson's disease, + strokes (atherosclerotic type is MC type of stroke), * subdural hematomas (falls cause tearing of bridging veins in subdural space leading to a venous clot), + sensory changes- * macular degencration (MCC of blindness in elderly), « skin disorders * UVB light-induced cancers, * actinic (solar) keratosis (common USMLE picture, precursor of squamous cancer), © endocrine system- type Il diabetes mellitus Potential for blindness, » limb hypoplasia, 15 : This material is copyrighted. All rights reserved. Edward Goljan, M.D. 2002 Top 5 causes of death in males and females regardless of age and sex i denendlag ‘order: * heart disease, « cancer, © stroke, » Sate shame we neuet Shienin (Corb), « motor vehicle accidents~ MC COD in males and females between 1-39 yes or ge Top 3 causes of death in children aged 1-14 yrs in descending order: * accid see MVAs, « falls 25-30%, » bums 10-15%), « cancer, « congenital anomalies Se Top 3 risk factors leading to increased morbidity/mortality in the United States in desce order: « cigarette smoking, * dietary factors and activity patterns: + ding diet, + lack of exercise, « aleohol abuse 'y partcms: « high sanurated fat, low fiber Gunshot wounds: « contact wounds~ contain soot and gunpowder in the wound (called fouling), intermediate wounds powder tattooing (stippling of skin around the entrance site), « distant wounds no powder tattooing, * exit wounds are larger and more irregular than entrance wounds Motor vehicle accidents (MVAs): * MCC accidental death between 1-39 yrs of age: commonly alcohol-related, particularly in teenagers, « seat belts and air bags have reduced morbidity/mortality Drowning: « common COD in children from I-14 yrs of age, + near drowning: survival following asphyxia secondary to submersion, © wet drowning: * 90% of cases, * initial laryngospasm on contact with water> rclaxation/aspiration of water, * fresh/salt water drowning: whether fresh or salt water drowning, surfactant is destroyed in lungs-» atelectasis with intrapulmonary shunting» diffuse alveolar damage and initiates spasm in the bronchioles, immediate COD in drowning is cardiac archythmia Burns: « first degree bums: * painful partial thickness bums (¢.g., sunburn), * produce ceil necrosis limited to the epidermis, + heal without scar, + second degree bums: + painful partial thickness bums, * iavolve entire epidermis,* form blisters within epidermis, heal without scarring unless they are deep, © third degree bums: painless full thickness bums, + extensive necrosis of epidermis/adnexa, * extensive scarring complicated by keloid formation (propensity for squamous cell carcinoma), * healing comes from residual epithelium at the margins of the bum and from adnexal structures, « COD: infection due to most commonly to Pseudomonas aeruginosa followed by Staphylococcus aureus, * house fires: * smoke inhalation MC COD, + carbon monoxide (CO) and cyanide poisoning commonly occur (source of cyanide is polyurethane in upholstery) Heat injuries: + hyperthermia: core body temperature >37.2°C, heat cramps: * afebrile, * common in untrained athletes/laborers who become volume depleted, * lose excess amounts of saltwater, « heat exhaustion * mild elevation in core body temperature (539°C), * common in athletes training in ho/humid environment, * develop severe volume depletion, * exertional type of heat stroke: # core body temperatures 2 41°C, + people working or running on het day, * skin hovdry, * profound lactic acidosis, + rhabdomyolysis common, + non-exerti stoke- * elderly/chronically ill patients, * skin hot/dry without sweating (hypohidrosis), * lactic acidosis/rhabdomyolysis uncommon, « malignant hyperthermia~* AD disease, * defect in calcium release channels in the muscle sarcoplasmic reticulum, + massive muscle contractions with extremely high temperatures after induction of anesthesia by halothane and succinylcholine (muscle relaxant), * Bx with dantolene, * screen family members with muscle biopsy and caffeine/halothane contraction test on muscle, * body temperature and redness of skin for patient walking briskly on a hot day- + no increase in rectal temperature, + vasodilatation of vessels in skin producing redness, * choice D in schematic, marathon runner on a hot day~ * increase in rectal temperature (probably heat exhaustion), * vasodilatation of vessels in skin producing redness, * choice Cin schematic AB c Rectal T A D Vasodilatation 16 9 44 q q 4q Note: This material is copyrighted. All rights reserved. Edward Goljan, M.D. 2002 Cold injuries: « frostbite: + localized tissue injury secondary to direct (ice crystallization in eels) cells) and indirect damage (vasodilatation/thrombosis), + tissue eS B hypothermia: * core body temperature <35°C, « tt exposed te teecag nig prolonged period of time, * uncoupling of oxidative phosphorylation, + venous pooliny wae Progress into circulatory failure and death ig that may Electrical injury: + Ohm's Law: ¢ current (1, amps) = voltage (E)/resi: 6 EE (HULGNR (olin, Semceat emmase knpeRiant seaerin eee temo direct current (DC) risk for electrocution, «resistance: * dry sin has the highest tissue resistance to current, particularly the hands and feet, * wet skin lowers the resistance to current, since voltage ig a constant, lowering resistance increases current, « ing from the left i is most dangerous- * involves the heart-> ventricular fibrillation, « blood is an excellent conductor of current, « COD: cardiorespiratory arrest with ventricular fibrillation and respiratory paralysis Jouizing radiation: * cxamples- y-rays, x-rays, particulate radiation emitted by radioactive substances (@/B particles), shorter the wavelength the greater penetration- + low penetration include /3-particles, * high penetration include y-rays, « radiation injury: direct or indirect injury to DNA, * indirect type produces hydroxyl free radicals from hydrolysis of water in the tissue, = DNA is most susceptible protein, » tissue susceptibility to radiation + directly related to degree of mitotic activity with the greatest effect on actively mitosing cells, + indirectly related to degree of tissue specialization (see below), * cell cycle— * peak sensitivity is in G, (synthesis of tubulin for mitotic spindle) and M phase (assembly and disassembly of mitotic spindle), * § phase Icast sensitive, + G, phase intermediate sensitivity, © tissues with a high radiosensitivity— * hematopoietic cells are most affected (e.g., lymphocytes > granulocytes > platelets > mature RBCs), * germinal cells in the testes, ovaries, « tissues with low radiosensitivity « brain is most resistant to radiation, * bone, * mature cartilage, * muscle, « total body irradiation: + hematopoietic system first system affected (greatest overall effect» lymphopenia (first change)» thrombocytopenia» bone marrow hypoplasia), « vessel changes thrombosis (early), fibrosis (late, ischemia), * skin changes including erythema cdema- blistering chronic radiodermatitis> potential for squamous cell carcinoma, * GI tract with diarrhea Ultraviolet light (non-ionizing radiation): « UVA- Wood's lamp (black light), ¢ UVB- + sunbum, * corneal bums from skiing, * mutagenic effect on skin (thymidine dimers), « UVC- germicidal, * UVB-related cancers- * basal cell carcinoma is MC skin cancer, * squamous carcinoma (precursor is actinic/solar keratosis), * malignant melanoma Laser radiation: intense localized heat that is equivalent to a third degree bum Microwave radiation: « produces a skin bum, * adversely affect pacemaker devices, * inconclusive complications— * cataracts, * cancer, + sterility Infrared radiation: * bums, + cataracts Bligh altitude it general- * O, concentration is 21% (normal), * decreased barometric pressure (e.g. only 200 mm Hig on top of Mt. Everest), + lation is essential at_high altitude since it lowers alveolar CO (respiratory alkalosis) > automatically increases alveolar PAOp-> increases arterial PO2, © acute mountain sickness: * occurs within the first 24-36 hrs of an ascent above 8000-10,000 feet, + headache, lethargy, insomnia, dyspnea, + Rx with descent, increase fluid intake (increased insensible water loss from mucous membrane evaporation), and ‘oxygen, * prevention by acclimatizing before ascending and using acetazolamide, which is 2 carbonic anhydrase inhibitor that produces metabolic acidosis compensation for the expected respiratory alkalosis, » high altitude pulmonary edema: non-cardiogenic Changes after death: rigor mortis is due to decrease in ATP in muscle Sites of alcohol (ethyl alcohol, ethanol) reabsorption: ¢ small intestine- 75%, » stomach + 25%, « partially metabolized by alcohol dehydrogenase 17 Note: This material is copyrighted. All rights reserved. Edward Goljan, M.D. 2002 = Liver metabolism of alcohol: alcohol dehydrogenase aldehyde deh ‘ Sige at relia Wate” R me eee aN) Pharmacologic action of alcohol: + CNS depressant in descending order> oe seenylcon limbic system-» cerebellum-> lower brain stem, * potentiates inhibitory neurowansmittere He aminobenzoic acid, * aldehyde dehydrogenase deficiency: * affects ~40% of Asians, = aie ays acetaldehyde and GI upset (similar to action of disulfiram), * alcohol toxicinws sy ne euphoria, gregarious, + 100 mg/dL, legally crunk in most states (slurred ee mat speech, uncoordinated), + eens mg/dL. stupor or coma, * >500 mg/dL. death, + delirium temens- « following occur 3-5 ys aftr complete witraval-» wemblousnes, disorientation, visual hallucinstions, agaton, + Diseases where alcobol is the leading cause: © thiamine deficiency: o Korsakoff’s psychosis, congestive cardiomyopathy, « macrocytic eee ieee acquired sideroblastic anemia— microcytic anemia with ringed sideroblasts, « Mallory Weiss syndrome: tear of the distal esophagus/proximal stomach from retching, + Boethaave's syndrome: rupture of the distal esophagus/proximal stomach from retching, © cimthosis, « esophageal varices— effect of portal vein hypertension due to alcoholic cirrhosis, « fatty change in the liver, » hemosiderosis— alcohol increases the reabsorption of iron, * acute and chronic pancreatitis, © type IV hyperlipidemia alcohol increases synthesis of VLDL, + Klebsiella pneumoniae pneumonia ‘Alcohol as a cancer risk: squamous carcinoma (synergistic with smoking): oropharynx, mid- esophagus, larynx, » adenocarcinoma: pancreas, liver Alcohol effects on CNS/PNS: + Wernicke’s syndrome/Korsakoff's psychosis, « cerebellar degeneration: Hu and Yo antibodies noted in spinal fluid, « dementia, « DTs, » distal peripheral neuropathy, © central pontine myelinolysis: demyelination syndrome due to rapid IV Rx of hyponatremia Smoking epidemiology: * MCC of premature death in the United States, * MC single preventable cause of cancer, * incidence of smoking is increasing in women and decreasing in men, nicotine intake can be monitored by measuring plasma or urine level of cotinine~ cotinine is only derived from the metabolism of nicotine, « MOA of nicotine: + absorbed rapidly into the pulmonary circulation, * moves into the brain where it attaches to nicotinic cholinergic receptors to produce its gratifying cffects/complication of smoking, * highly addictive agent, + inactivation of the p-53 suppressor gene by a point mutation on chromosome 17 is the MC genetic defect in smoking induced cancer Cancers where smoking is the leading cause: ¢ hmg cancer: squamous, small cell, and adenocarcinoma to a lesser extent, * MCC of death due to cancer in both men and wornen, * oral pharyngeal and laryngeal and mid-esophageal squamous cancer, * pancreatic adenocarcinoma, © transitional cell carcinoma of bladder, + renal adenocarcinoma, Cancers where smoking has been implicated but is not the MC risk factor: » cervical cancer~* squamous cancer, * carcinogens found in cervical secretions, * stomach adenocarcinoma, « breast adenocarcinoma primarily in women who are slow acetylators of N-acetyltransferase 2 enzymes, * prostate adenocarcinoma, * colon adenocarcinoma, * leukemia~ increased risk of both lymphoid and myeloid leukemias, » alcohol is a cocarcinogen with smoking that further enhances the risk of oropharyngeal, esophageal, laryngeal cancers, ¢ smoking + asbestos exposure markedly enhances the incidence of primary lung cancer no association of smoking with mesothelioma, whether the patient is a smoker or not, lung cancer is the most common cancer associated with asbestos exposure 18 Note: Ths materia s copyrighted, All igs reserved. Btward Goljau, M.D. 2002 7 Smoking effects om the cardiovascular/CNS recurrent AMI as well, increases risk for sudden cardiae death, vascular disease, + increases risk for strokes, « contributing factors, to chemicals in smoke and nicotine effect on blood pressure and fy Profile, + tssus hypoxia secondary to excess carbon monoxide (CO), = Smoking effects on the respiratory system: « COPD. 80% ey all i e 4 of all cases, chronie bronchi Emphysems, » recurrent infections--Pocamonia, URIs, «exacerbates bronetny abe cee 7 Smoking effects om the Gl system: GERD, «delays the rate of uses heaigg Fisk for oral, upper and lower Gil cancer, « USMLE scenario advice to piven Rismryot ant, ulcer disease: stop smoking = sevoker th Hits 7 Effects af smokeless tobacco (enuff, chewing tobacco): * nicotine addion leukoplakia/cancer— inside the lip, under the tongue or cheek. « vernicous sausmaus canes cea cancer snuff users, « aggravation of cardiovascular disease~ nicotine oats Smoking effects on bone and menopause: » increases the risk for ssteoporosis in men and women, # biochemical reaction in women— * estradiol (most metabolized in the liver into estrone, which is metabolized into AaMIO) oF estriol (strong estrogen actvity)-> smokers have greater conversion of ouene nee an, inactive metabolite leading to tow estriol levels-> low levels of estrogen incieace the nue we osteoporosis and premature menopause ‘* Passive smoking effeet on children: « pathogenesis of passive smoke effects~ * ~ 75% of tot combustion product in a cigarente is exhaled, » risk of passive smoke extends to children as well adults, « increases the incidence of SIDS, » increases risk for lung canccr- 1-2 times inmeacea ases risk for otitis media, * increases risk for recurrent upper and lower respiratory infections Miscellaneous smoking effects: + increases risk for developing proteinuria in diabetes mellinis, directly responsible for ~25% of residential fires, ¢ vitamin C deficiency 2 Beneficial effects smoking cessation: « longevity— smokers who quit before 50 yrs of age have half the risk of dying over the next 15 ‘yrs than a smoker has, * lung cancer- + in 10 ‘yrs, there is a 50% reduction in lung cancer when compared to a smoker, after 15 yrs, there is only a 16% risk for Jung cancer when compared to a smoker, « AMI- AMI risk approaches that of a nonsmoker after | y# of abstinence, * pregnancy— pregnant women who stop smoking in the first trimester reduce the tisk of a low birthweight baby to that of a nonsmoker, « forced expiratory volume in 1 second (FEV) * it is not improved by cessation of smoking, * rate of decline is similar to that of a non- smoker Drugs and interstitial pulmonary fibrosis: * amiodarone, « bleomycin, © busulfan, « cyclophosphamide, » nitrofurantoin, « nitrosourea, * methysergide— also retroperitoneal fibrosis and Raynaud phenomenon, « methotrexate, + procarbazine Occupation exposure relationships: + automobile mechanic carbon monoxide, * pesticide industry- * organophosphates, * arsenic, ¢ meat packing— polyvinvl chloride with risk of hepatic angiosarcoma, + insulation/demolition/roofing material- + asbestos: lung cancer, mesothelioma, fibrous pleural plaques (MC overall complication of asbestos), + formaldehyde, dry cleaning carbon teirachloride with liver necrosis due to free radicals, « rubber/chemical industry * benzene: aplastic anemia, leukemia, + aniline dyes: bladder cancer, « battery, smelter, plumber/foundry— lead poisoning, painter- + methylene chloride: converted into carbon monoxide, * solvents, * lead, © petroleum- * benzene, + polycyclic hydrocarbons: lung cancer, « sewer worker~ hydrogen sulfide gas: sulthemoglobinemia Isopropyl alcohol (rubbing alcohol) poisoning: ¢ metabolism * metabolic end-product in the liver_is acetone: no metabolic acidosis unlike other alechols, + increases serum osmolal gap: 19 VETHe 4 G44 q q differmnee between calculated and measured serum osmolality >10, « etinial- Methyl alcohol (Wood's alcohol): © increased anion i formic acid), © optic neuritis and potential for blinnicans Toke v Sins Converted into competitive antagonist with methyl alcohol for alcohol dehydrogenase ANCE: ethancl is a Ethylene glycol (antifreeze): « increased anion gap metabolic acidosis (con * renal failure from calcium oxalate crystals obstructing the lumens, « Rec is a competitive antagonist with methyl alcohol for alcohol dehydrogenase Mercury poisoning: * MOA- * toxic in inorganic (elemental) fori: dental amalgan used in hat making industry (‘mad hatter disease"), + toxic in organic fom oo contaminated fish, + clinical~ + diarrhea, * visible on x-rays, + nephrotoxic ATN mene broximal tubules, + cerebral/cerebellar neuron loss, * constricted visual fields, = Rec deseo Arsenic poisoning: * sources + pesticides, + animal dips, « Fowler's solution, » Ra af sey 1930's, « clinical * garlic odor to breath, » severe diarthea: "rice water" stools similas ie ele seni melanosis: wry in with dark macules, « Hl carcinoma o} = nails have transverse (Mees nails): concentrates in keratin/hait/nails, © j ; MC COD, * nephrotoxic acute tubular necrosis involving proximal alae lieve ar Rx- dimercaprol NSE Saeiosarcoma, Mushroom poisoning (Amanita): « MOA— toxin inhibits RNA polymerase, © : pain/vomiting, « bloody diarthea, « jaundice: extensive fatty change re eee Petroleum product (gasoline, Kerosene) disorders: » euphoria (drunk acting) when inhaled (or ingested), » addicting, « toxic doses— + convulsions, + tinnitus, « non-cardiogenie pulmonary edema Strychnine poisoning: « MOA- CNS stimulant that blocks postsynaptic inhibition, clinical (Similar to tetanus)- + tetanic convulsions, + opisthotonus, + risus sardonicus, + death Poisonous snake envenomations: = types~ + pit vipers: rattlesnakes (MC bite), water moccasins, copperheads, * true cobras: coral snake (neurotoxin that blocks acetylcholine release-> paralysis and death, similar to botulinum poison), coral snake has following color banding “red and yellow Kill a fellow", harmless scarlet king make~ "ted and black friend of jack", » pit viper cavenomations~ + local swelling/necrosis, + hematologic problems: DIC, + antivenin is available: danger of serum sickness CDyo: common acute lymphoblastic leukemia antigen (CALLA) Kid staring into space that has broken bones: absence seizures Testicles in hernia sac: * testicles are not translucent, * translucency in the scrotal sac implies a hydrocele Dock worker for 20 yrs with lung mass: primary jung cancer secondary to asbestos exposure from asbestos insulation around pipes in ships Belly full of scars: acute intermittent porphyria due to deficiency of uroporphyrinogen synthase Flame hemorrhages ia the retina: sign of hypertensive retinopathy Visual field defect with craniopharyngioma: bitemporal hemianopsia (impinges on optic chiasm) Stacks of hemoglobin when it the blood is deoxygenated: HgbS Patient with normal PT, PTT, bleeds after surgery even after receiving fresh frozen plasma: probable platelet problem~ ¢.g., thrombocytopenia, patient on NSAIDs Cavitary lesions in upper lobe: « MCC is reactivation TB (not primary), + histoplasmosis, * Klebsiella pneumoniae, squamous carcinoma . Well-differentiated squamous cell carcinoma of lung or any other tissue: * keratin is red, ¢ look for squamous pearls Patient with Marfan's syndrome has diastolic murmur: aortic regurgitation due to stretching of aortic valve ring by the dissection deep coma with verted into oxalic acid), with TV ethanol: ethanol 20 Note: w8 oy ¥¢ 994Gq 4 q a4 ‘This material is copyrighted. All rights reserved, Edward Goljan, M.D. 2002 Xray Gindings in osteogenic sarcoma: mass in met i % muscle, * upper tibia or lower femur, + x-ray~ « Codman'e eee invasion imo surrounding made by osteoid, « Rb suppressor gene relationship + * sunburst appearance of bone X-ray fladings ia Ewing's sarcoma: onion skinning around bone shaft Young woman with episodic hematuria after upper res absent uroporphyrinogen synthase, « belly full of scars Mone ‘with maculopapular rash during pregnancy and neonate with saber shins congenital Baavte infarct of brain in patient with atria! fibrillation: embolic infarct "atient with twitching of face with tapping of facial nerve: « tetany calcium threshold potential is (GE ca Blears are partially sea eee ‘gn, * Trousseau’s sign is when thumb adévcts into palm when taking blood preseare joe ares not develop ‘ = transfusion reaction when given wrong ABO roup: elderly patients normal} isohemas attack BOantger ineckootml ieslutnins with age, so none may be presentto Screen for autoimmune disease in relatives: screen for HLA antigens unique to the autoimmume disease or serum ANA depending on the way the question is worded Source of Legicrelia infections: + air conditioning cooling tawersicondensers, * showers, « vegetable misters in grocery stores, + produces interstitial nephritis with type TV renal mibulr acidosis: desuruction of JG apparatus with hyporeninemia and kypoaldosteronicen Patient taking penicillin for streptococcal infection develops hemolytic anemia: due to sutoimanune hemolytic anemia (type TI) against BPO attached to RBC membrane Epidural hematoma: » fracture of temporoparictal bone and rupure of middle meningeal arery Malabsorption with blunt celiac disease, « order anti-gliadin or endomysial antibodies EK disease from brain instrument treated with formaldehyde: prions are the infective agent “Bronze diabetes”: hemochromatosis 40 yr. old man with dementia and senile plaques in brain: Down syndrome patient with Alzheimer's Easy bruising in hospitalized patient on antibiotics: vitamin K deficiency from destruction of colon bacteria by the antibiotic Pancreatic tumor associated with peptic ulcers: ZE syndrome with secretion of gastrin Multigucleated giant cell associated with viral infection: measles~ Warthin-Finkeldcy giant cell Mechanism of cerebral edema in Pb poisoning: « increase in 8-aminolevulinic acid in brain is toxic and produces demyelination and increased vessel permeability, can be prevented by taking heme, which inhibits ALA synthase Alcoholic with bad breath and cavitary lung lesion: + lung abscess from aspiration of ‘oropharyngeal material, * mixed aerobes and anaerobes Main site for acetaminophen toxicity in liver: « around terminal hepatic venule (centrilobular) least amount of oxygen in this site (zone IM), « also the site for fatty change in alcoholic or shock Biock left renal vein: © produces a left-sided varicocele, + left spermatic vein normally empties into the left renal vein mechanism for varicocele, « the right spermatic vein ermpties into TVC Injury to head with polyuria: central diabetes insipidus from transection of pituitary stalk Supine and where does foreign body go: superior segment of RLL 21 Note: This material is copyrighted. All rights reserved, Edward Goljan, M.D. 2007 va 994 a4 a4 In a patient who is lying ou the right side, a foreign bod: commonly localize to these primary sites in the nag SHER a penterioe segment middle lobe, » right upper lobe [a a patient who is lying on their back, aspirated a ‘ segment of the lung: right lower lobe superior segment VOUE Most likely localize to this Lymph node: « B cells in follicles, « T cells in parafoliicul . Patient with an increase in amylase and lipase: « pation mase ReneS Siuses Pancreatitis, « cannot be mumps because of lipase Mely an alcoholic with acue Staghorn calculus: « magnesium ammonium hate, = alkaline urine pH Phosphate, © urease uropathogen like Pro‘eus, « Overdose of barbiturates: acute respiratory acidosis Bye effects in Cushings syndrame: cataracts from increased glucocorticoids Flank mass in a child: © Wilms tumor, « associated i 4 ia ine ssociated with aniridia and. hemihyperrophy in the AD What makes a laboratory test more specific?: raise the upper I orm i specificity (less FPs) and positive predictive value per limit of normal, which increases Findings at autopsy of RDS: * hyaline membranes with massi ys eaBloctas eats massive intrapulmonary shunting ive atelectasis, » atelectasis leads to Baby bleeds after circumcision on 10th day in a mother who i Baby Deeds atts: dr is breast feeding her baby: no Abetalipoproteinemia: + absence of apo B leads to low CH levels, « malabsorption, « all lipid fractions, + blindness, « hemolytic anemie, «treated with vitamin PN EEO Sudden onset left flank pain, hypotensive, pulsatile mass: i Saddea sunita sib puls Tuprured abdominal aortic ancurysm palpitations particularly when anxious: + mitral valve prolapse, + myxonatous degeneration due to an increase in dermatan sulfate Click murmur relations in mitral valve prolapse: « dectease preload causes click and murmur to come closer to SI- + anxiety, * standing up, * Valsalva, « increase preload causes click and murmur to come closer 10 $2- + lying down, + clenching fist (increases systemic vascular resistance and decreases cardiac output), squatting (increases systemic vascular resistance) Aortic stenosis: MC valvular lesion associated with~ + hemolytic anemia with schistocytes, syncope with exercise, * angina with exercise 28-year-old patient has a family history of sudden cardiac death at a young age: hypertrophic cardiomyopathy Chambers or vessels with the highest SaO, in Tetralogy of Fallot: « pulmonary vein, « left atrium Chambers or vessels has the lowest SaO, in a newborn with a machinery murmur (PDA): « right atrium, « right ventricle Shuats or groups of shunts cardioprotective in Tetralogy of Fallot: « patent ductus~ unoxygenated blood goes to PA for oxygenation in lungs, « atrial septal defect— step up of oxygen in the right heart Viral myocarditis, develops hypotension, ueck vein distention, a drop in blood pressare 03 inspiration, and muffled heart sounds: pericardial effusion from coxsackie myocarditis~ Bx of heart would show a lymphocytic infiltrate with destruction of muscle First step in management of pericardial effusion: echocardiogram Newborn baby girl, swelling of dorsum of bands/feet (lymphedema), cystic mass in neck (dilated lymphatics that stretch skin and produce webbed neck): + lymphedema in child with Tumer's syndrome, + XO, « defects in lymphatics 22 yr old, ADDS, raised, red lesions on hard palate: + Kaposi's sarcoma, « most common site for KS in GI tact, due to fellatio, « Rx with intralesional o-interferon ertal would most - umiterial Is copyrighted. All rights rererved. Edwarg Goljan, M.D. 2002 ‘$8 yr old smoker, painless jaundice with clay colored stool! in z urobilin in stool), painful varicosities in ‘ight upper Shoe aren sat ee fem i 2 few weeks ago: » migratory thrombophlebitis ine patient with carcinoma of the head of ete et with obstruction of bile flow, called Trousseau's sign ofpancreas Patient with diarrhea and episodic flushi, "16 yr old died suddenty playing basketbat, i hypertrophy of the interventricular seprum with conduction defects, (worse) with factors decreasing preload (standing, Valsalva, venodilator), « murmur intensity Gecreases (better) with factors increasing preload (Sustained hand a blockes) SAP, squatting, lying down, B. ‘& Pulmonary capillary wedge pressure readings (normal < 12 mm Hg) expect bypovol shock (volume depletion) versus an acute myocardial infarction ino ee eae (increased hydrostatic py is neuroblastoma, © an APUD 2ye old girl with necrotic, bloody, grapetike mass reveals malignant cells with cross-striations; tumor si NTL related antigen, and carcinoembryonic antigen, and positive for desmin; cancer is most likely derived from: « muscle embryonal shabdomyosarcoma, * MC sarcoma in children * 6S-yr old woman with abdominal distention, Primary cancer most likely responsible for induration in pouch of Douglas on rectal exzmuinatica, overian cancer induration due to seeding by the cancer 7S-yr old man with point tenderness in the lower vertebral column, elevated serum alkaline Phosphatase (osteoblastic metastasis), tests ar procedinc would be your first step in the cvaluation of this patient: digital rectal exam think cheap, + bone tenderness implies stage IV disease and DRE should be positive Metastatic adenocarcinoma in left supractavicular node (called Virchow's node) would most Ukely have its primary origin in: stomach adenocarcinoma ‘* 1S-yr old girl, sudden onset of right lower abdominal Pain, pregnancy test negative, cystic paass with bone and calcifications in the right ovary; cystic teutone * Pediatric cancers into the order of decreasing frequency: acute lymphoblastic leukemia—> primary CNS tumors—> Burkit’s lymphoma» neuroblastoma = Most comunon ionizing radiation-indaced cancer: acute leukemia An ulcerated lesion that develops in a keloid Secondary to third degree burn or an ulcer tains negative for cytokeratin, factor ———--~ located..at the orifice of » chronically raining sinus that does not respond to medical Hanaeement is most likely due to: development of a squamous cell carcinorms ‘* Hematuria in a S8-yr old smoker would mast likely be associated with which of the following TrouPs of cancers: renal adcnocarcinoma/transitional cell carcinoma of the blades * Most responsible for the increased incidence of basal cell carcinoma ta the United States: sun exposure beginning at an early age ; % — Exudative lung reaction associated with exposure to moldy hay: Farmer's lung~ thermophi actinomycetes 23 ee ee ee ee * Young man presents with hemoptysis and then develops acute Goodpasture's syndrome, + anti-basement membrane antibodies, « type I ie . © — Hypersensitivity pneumonitis that is primarily seen in textile workers: byssinosi 7 Bypersensitivity pneumonitis that commonly occurs in farmers who enter a a it fermenting corn: silo filler's disease— nitrogen dioxide room with F _ Pneumoconiosis whose most common lesion is a benign pleural plaque: asbestosis A pregnant woman in her 32nd week of gestation has following drugs should she be given to protect her aby from develogiey aie eee ot te distress syndrome: glucocorticoids increase surfactant eynthesis in the baby ey as a yt old woman with gheenls Lesiichns develops episodic asthmatic attacks. The most cause is: ingestion of non-steroic angi . iat i-inflammatory agents with release of leukotrienes_ © Farmer and his wife are brought to the ER by their son because they are too weak to walk or drive and their vision is blurry and exam reveals ptosis, facial weakness, nonreactive dilated Pupils, dry mucous membranes, and normal DTRs—? diagnosis: « C. bonulinum food poisoning, Rx with botulism antitoxin, danger of respiratory paralysis toxin blocks the release of acetylcholine * Young woman with intermittent houts of diarrhea and constipation associated with cramping right and left lower quadrant pain: + irritable bowel syndrome— intrinsic motility defect in bowel, « flexible sigmoidoscopy is negative Intravenous drug abuser in prison—?type of hepatitis: HBV with outbreak of hepatitis: HAV most likely Hepatitis associated with urticaria, fever, arthralgias, and the nephrotic syndrome: « HBV, « serum sickness type of disease~ type IN immunocomplex mechanism, + vasculitis associated with polyarteritis nodosa = Immunizations given at birth to a baby whose mother is positive for HBsAg: * Hep B vaccine- active immunization, + HBIG- passive immunization = — Ina patient with By: deficiency who is being treated with pharmacologic doses of folate, which of the following will be corrected; * neurologic deficits remain, hence the importance of making the correct diagnosis, ¢ megaloblastic anemia is corrected © Im an alcoholic with a macrocytic anemia and hypersegmented neutrophils and a normal neurologic exam, which of the following tests is most indicated: RBC folate is more sensitive than serum folate * A 4 yr old child has eaten rat poison and is hemorrhaging: « rat poison is warfarin, which blocks all the vitamin K-dependent factors, « both PT end PTT are prolonged, © Rx with IM vitamin K * — Child living with elderly grandparents develops a GI bleed: child ate grandparents warfarin = A child has eaten raw hamburgers and now has a hemolytic anemia and renal failure: « HUS due to 0157: H7 serotype of E. coli, + low platelet count and prolonged bleeding time A woman has menorrhagia, easy bruising, and epistaxis: « VWD, « prolonged bleeding time and prolonged PTT = A man bas a family history of a bleeding disorder which began with bis mother's father: + hemophilia A, + matemal father transmits the disease to all his daughter's (SXR trait) who are asymptomatic carriers, « the daughters transmit the gene to 50% of their sons, prolonged PTT © A449 yr old woman with fibromyalgia is taking NSAIDS for pain: qualitative platelet defect, « prolonged bleeding time Major crossmatch: patient scrum reacted against donor RBCs, * does not guarantee survival of the infused donor RBCs, * does not prevent patient antibodies developing against donor RBC antigens, « detects the presence of patient antibodies against donor RBC antigens Note: This material is copyrighted. Allright reserved. Edward Goljan, M.D. 2002 A phlebotomist inadvertently sticks himself wit with AIDS. Which of the following infections is ¢) 1:300 chance of becoming HIV positive A phiebotomist inadvertently sticks himself with a needle after drawing b Which of the following infections is the phlebotomist at most risk Sireteag ee the greatest ial load in blood ofall vines, «you cannot get syphilis fom tance bee Y Te Most common antibody encountered in clinical practice: it alse impossible to find bi that is negative for antibodies against CMV ee ‘Hemorrhagic skin necrosis associated with warfarin therapy: BY: heterozygote C become homozygote when given warfarin in ~6-8 his when the enn ccarboxylated protein C disappears now the patient is hypercoagulable A patint who bas chronic bepatids and has been transfused ia the past most Likely has antibodies directed against: HCV is the MCC of transfusion henesit 28 yr old woman bas an anterior mediastinal mass and non tender lymmphad. Fight supraclavicular node: nodular sclerosing Hodgkin's disease, ™PNAG@ROPatby In the An afebrile 72 year old man with won-tender, generalized lymphadenopathy hepatosplenomegaly, normocytic anemia, thrombocytopenis and 1iypticieglhelioene! most likely has: « CLI is. B cell malignancy, « neoplastic B cells cannot transform into plasma cells Rationale for beginning heparin and warfarin at the same time: © the shortest are factor VIO and protein C and the langest is prothrombin, = previously activated vitamin K-dependent factors have long half-lives A.7S year old woman in a nursing home has nou-palpable ecchymoses limited to the back of her hands. A CBC is unremarkable, The patient most likely bas: senile purpura Most common type of hereditary thrambosis disorder: factor V Leiden cannot be degraded by protein C and S Most common infection transmitted by a blood transfusion is due to: « CMV is Present in donor lymphocytes, + radiation of the blood kills the lymphocytes and prevents infection in the recipient « bone marrow transplant patents are particularly prone to CMV infections in the lings Which of the following vitamin toxicities predisposes a patient who is taking warfarin to bleed and have an INR outside the normal range: vitamin E toxicity decreases the synthesis of vitemin K dependent factors in the liver A weightlifter develops a sudden onset of 2bdominal pain along with bypovolemic shock. At surgery, his abdominal cavity is filled up with blood. The cause of the intraabdominal bleed is most likely associated with: anabolic steroids cause liver cell adenomas, which have 2 tendency to bleed A 30 year old man with acquired immunodeficiency syndrome has pitting edema, hypertension, proteinuria > 3.5 g/24 brs, and fatty casts in the urine. A renal biopsy exhibits glomerular disease: focal segmental glomerulosclerosis A 29-year-old woman presents with generalized pitting edema and hypertension. A renal biopsy reveals diffuse glomerular disease with increased thickness of basement membranes and bypercellularity. Silver stains reveal "tram track" splitting of the basement membranes. An electron micrograph exhibits "dense deposits” in the glomerular basement membranes. The patieat has a persistently low serum C3: * type II MPGN or “dense deposit disease”, Patients have C3 nephritic factor- autoantibody against C3 convertase in alternative system that causes it to continually activate C3 causing very low levels A 25-year-old man, who initially presented to the hospital with hemoptysis, has progressed into renal failure. A renal biopsy shows linear immunofluorescence.: Goodpasture's syndrome patient has rapidly progressive crescentic glomerulonephritis, 2 needle after drawing blood from a pati ¢ phlebotomist at most risk for contracting: 25 A 74-year-old man with colon cancer develops generalized an: . 24 by aed Cutty enets fn hiscorine. A renal biopsy shows dite gloat > 3.5 stains demonstrate epimembranous spikes and the presence of subepithelial wena electron microscopy: diffuse membranous GN leposits. on A 10 yr old boy living in Salt Lake City, Utah bas bilateral sensori 4 ir ity ig Su Lae CD, Ua bs ha sence ot visceral epithelial cells: Alport's syndrome sex-linked dominant disease notedin A65 yr old man with chronic HCV hepatitis has a history of cyanosis of hi suid cars durtug cold weather: These findings subside when tae comes betoora ee palpable purpura in cold-exposed areas: cryoglobulinemia s A 12 year old bas a history of cellulitis ~2 weeks ago. He now pr: Vl pectoris byperdcadiem, and sueley colored uring, Urinslpsi shows Ge casts Chl ce (15 g/24 h). "Renal biopsy reveals a diffuse increase in cellularity along with a neutrophilic inflerate, IF shows granatar deposits Subepithelial deposits are noted on EM: acute pot streptococcal GN- immunologic reaction secondary to either ee or skin infection may 3 group A strep Pharyngitis A 62-year-old woman with a long-history of severe rheumatoid arthritis and restrietive cardiomyopathy now presents with pitting edema and hypercholesterolemia. Urinalysis shows oval fat bodies and fatty casts. Protein dipstick is 2+, sulfosalicylic acid 2+. Renal biopsy shows hyalinization of the glomerular mesangium. A special stain is order: amyloidosis ‘An 89-year-old woman with a history of chronic HCV hepatitis from a blood transfasion now presents with generalized puffiness and mild hypertension. Urinalysis shows oval fat bodies, fatty casts with Maltese crosses, and 4+ protein (> 3.5 g protein/24 hr), A renal biopsy reveals hypercellular glomeruli with an increase in basement membrane thickness and tram-track splitting of the basement membranes. The IE is granular and EM reveals subendothelial deposits, The patient has depressed levels of C3: type I MPGN with HCV association A 24 year old man with a prior history of an upper respiratory infection is noted to have microscopic hematuria and mild proteinuria (1.5 9/24 brs) during a routine physical exam. Be is not hypertensive. A renal biopsy shows a granular IF with predominantly IgA deposition in the mesangium and electron dense deposits in the same area: ig glomerulonephritis : A 48-year-old worvan has proteinuria (> 3.5 gm/24 hrs), bypertension, and fatty casts in the urine, A renal biopsy reveals eosinophilic nodular masses in the mesangium of the glomeruli, hyaline arteriolosclerosis of the afferent and efferent arterioles, and increased thickness of the basement membranes of the tubules. IF is negative. EM reveals fasion of the podocytes and increased collagen deposition in the mesangium and basement membranes: nodular glomerulosclerosis in diabetes mellitus A 28 yr old man, with a history of renal disease in his maternal grandfather, has end-stage renal disease requiring dialysis. Physical exam reveals papular red lesions on his skin, peripheral neuropathy, and corneal disease. Previous renal biopsies exhibited vacuolated visceral epithelial cells which correspond with lamellar bodies on EM: Fabry's discase~ SXR disease with deficiency of a-galactocerebrosidase A and accumulation of ceramide trihexoside A normotensive 10 yr old boy, with a previous history of an upper respiratory infection, presents with generalized pitting edema. He has had problems with allergies since early childhood. Urinalysis reveals proteinuria (> 3.5 9/24 hrs), fatty casts, and oval fat bodies. His clinical findings improve rapidly with high dose corticosteroids: lipoid nephrosis A 25 yr old woman presents with hypertension. A urinalysis reveals mild proteinuria, hematuria, and RBC casts. A serum ANA exhibits a rim pattern. An anti-dsDNA titer is extremely high: type IV SLE glomerulonephritis |: systemic 26 Note: a4 te ‘Ths tera is copyrighted. All ights reserved, Edward Gaia, Mp. 2002 A child with Tarner's syndrome is noted to have an abo horseshoe kidney,» lower pole fusion,» behind the inferior meneie ene Patient with Wegener's granclomatosis develops hematuria seg + wansitional cell carcinoma, * patient is on eyclophomphaennns cystitis and TCC. " Ax Feyptian man bas microscopic hematuria and an abnormal atine cytology: squamous c arcnoma: patient has Schitzosoma hematobium involving the bladder viene 7 “ An uncircumcised male bas an utcerative lesion on the undersurface of the zlans peais and palpable inguinal adenopathy: « squamous cell earcinoma (penis) = hak of ‘ireunocision is greatest risk factor note the bilaterality A Pacene mith acute myelogenous leukemia developed acute renal failure after being, aggressively treated with multiple chemotherapy agents; « urie acid crystals « example of tumor lysis syndrome Crystal associated with ethylene glycol poisoning: calcium oxalate inborn error of metabolism associated wich renal stones and hexagonal crystals inthe urine: cystine eystinuria Gephlication associated with sickle cell trait, acate pyelonephritis, analgesic abuse, and diabetes mellitus: renal papillary necrosis Depression related to = vitamin deficiency: niacin deficiency comclation with decreased tryptophan conversion into serotonin Vitamin difference between ovo Inctovegetarian and 2 pure vegan: pure vegan lacks Bra, while ovolacto does not Farmer rubs lesion off from the back of his neck but it grew back: actinic (solar) keratosis Taborn error of metabolism associated with pellagra: Harmup's discasc~ « loss of neu amino” acids in the Gl and GU tact, * loss of ryptophan leads to pellagra due to decrease in synthesis of niacin MCC of increased plasma homocysteine in United States: folate deficiency Patient taking dapsone develops cyanosis resistant toy therapy: patient has methcmoglobinemia due to oxidation of iron to ferric state Difference of B° and B° thalassemia: f° synthesizes some B chains, while ° does not synthesize any B chains due to a stop codon Sequence in hypoxic cell injury: + hypoxia+ 4 oxidative phosphorylation in mitochondria leading toa 4 ATP—+ J ATP leads to: « T anaerobic glycolysis (1 intracellular pH from lactic acid, ¥ etycogen), * dysfunction of Na‘/K” ATPase pump (reversible cellular swelling), « ribosomes seach fiom RER (+ protein synthesis, fatty change) ~» 1, jmeversible cell membrane injury: « lnuacellular release of lysosomal enzymes damages membrane, « endogenous activation of Phospholipases (7 influx of Ca” into cytosol) with release of toxic lipid products, « cytoskeletal alterations (activation of proteases by Ca”), 2. imeversible nuclear changes: « activation of nuclear enzymes by Ca", = nuclear pyknosis and lysis) —> 3, reversible mitochondrial dysfunction: « entry of Ca™ into mitochondria with activation of phospholipases causing destruction of inner and ‘outer membrane, + Ca" produces large densities SLE glomerulonephritis: antibody excess immune deposits combined with IgG a Note: This material is copyrighted. All rights reserved. Edward Goljan, M.D, 2002 a4 a4 99 F949 G4 GE a qa aq a4 Patient with SLE with Raynaud's phenomenon: more common with PSS and CREST African American (ook primaquine and then developed an ae Gna x ped an anemia with bite cells: G6PD Patient with intense occipital headache and blood in CS) patient with a ruptured congenital berry aneurysm Septic arthritis in young adult: Neisseria gonorrhoeae Chitd with disease characterized by neutrophils that cannot ki ia: sganulomatous disease of childhood or myeloperoxidase defcieney nari Cou be chronic Thrombotic thrombocytic purpura: * platelet thrombi (not DIC), * CNS hemorhage, « hemolytic anemia with schistocytes, «renal failure, «thrombocytopenia (consumed in roving CSF analysis in AIDS patient with meningitis showed an encapsulated organism: crypioceccus Calculate anion gap: + AG ~ scrum Na* - (serum CI’ + serum HCO,), « increased AG (add atid) duc to lactate, salicylate, B-OHB, AcAc, oxalate (ethylene glycol), formate (methyl alcohol) Phosphate/sulfate (renal failure), ¢ normal AG (lose bicarbonate) due to~ diarrhea, renal tubula: acidosis (proximal and distal) Organism causing pyelonephritis: + E. coli, « ascending infection due to vesicoureteral reflux Febrile woman with non radiating flank paiu on right side. What would be present in her urine?: WBC casts (pyelonephritis) Woman with painless nodule lateral to cricoid cartilage: probable papillary cancer of thyroid MEN I: « pinuitary tumor, » parathyroid adenoma, + pancreatic tumor (usually ZE), + peptic ulcer (due to ZE and hypercalcemia from hyperparathyroidism) MEN Dla: « medullary carcinoma thyroid, * hyperparathyroidism, « pheochromocytoma MEN Ib: medullary carcinoma, « pheochromocytoma, * mucosal neuromas Calcitonin is marker for medullary carcinoma of thyroid: calcitonin lowers calcium by inhibiting osteoclasts ‘Bypercalcemia in a patient with MEN syndrome: hyperparathyroidism with increased PTH Conn’s (primary aldosteronism) syndrome: = key findings~ « hypertension, « no pitting edema, ‘* hypernatremia (mild or upper limit normal), * hypokalemia, + metabolic alkalosis Hypoaldosteronism effects (spironolactone, Addison's, destruction of JG apparatus): + hypovolemia from salt loss, + hyperkalemia, + normal AG metabolic acidosis Mutation producing neurofibromatosis: inactivation of NF suppressor gene Hepatic encephalopathy with flapping tremors: relates to false neurotransmitters and increase in ammonia from bacterial degradation of urea in the intestine Sexually active man with sterile pyoria and no organisms seen on gram stain: probably Chlamydia trachomatis non-specific urethritis Man with pain in posterior aspect of left testicle: « epididymitis, + depending on age of the patient organism could be Neisseria/ Chlamydia if < 35 or E. coli/Pseudomonas aeruginosa if > 35 Young woman with 2 stroke and irregular left ventricle filling: embolization from atrial myxoma Lady that just returned from Asia has severe dyspnea pulmonary embolus from sitting too long Cystic hygroma in the neck area in a fetus: + Tumers syndrome, « lymphatic defects are conimon— cystic hygroma becomes webbed neck, « lymphedema of dorsum of hands and feet Tumor of 4* ventricte: * ependymoma- neoplastic ependymal cells, « usually in children Ascites in a patient with 2 normal liver biopsy: portal vein thrombosis leading to portal hypertension Elderly woman with 2 days of eye pain and dilated vessels on white of the eye that don’t blanch with vasoconstricting drugs: probable glaucoma Atrophy of tail of caudate nucleus: Huntington's disease Vitamin D excess: « hypercalcemia, « urinary stones * probable subarachnoid bleed in a in a day of arrival: probable 28 ae qq 9494 9999gad 4 ‘Vitamin C excess: calcium oxalate stones Paget's disease of bone in an elderly woman: « early phasc of lysis (resembles multiple myel except it does not have clear margins) and then bone deposit seo: fractures), mereased seram alkaline phosphatase Gepesition with soft mosaic bone (pathologic Man with severe hyponatremia and high urine osmolality: » i small cell cancer in the lung, * high urine osmolality is ian eed patie Pe Prrarareand Joes of sodium in the urine as well as constant concentration of the urine from excess Factitious insulin injection: « hypoglycemia, « high serum insulin, ¢ low C-peptide by hypoglycemia), « if C-peptide is increased, patient has ‘nsanoma’ (orppressea Type 1 vs type II: « type J has insulitis due to autoimmune destruction, type I has amyloid in islets, « type Thas HLA relationship, type I has family history, « type I has antibodies against islets and insulin, type 0 has no antibodies, * type I is complete insulin deficiency, type HI is relative baal deficiency with decreased insulin receptors (increased plasma insulin) and postreceptor Post-transfusion hepatitis: HCV Pulmonary fibrosis with pulmonary artery hypertension: » could be duc to drugs (amiodarone, bleomycin, busulfan, methotrexate), « Hamman-Rich hung from alveolitis syndromes AIDS patient with purple lesions on skin: Kaposi's sarcoma due to Herpesvirus 8 Case study of child with medulloblastoma: malignant cerebellar tumor Alzheimer's disease: © remember Down syndrome relationship and chromosome 21: 40 yr old with AD is an adult with Down's syndrome, « senile plaques, * loss of higher intellectual function Man with extrapyramidal signs: Parkinson's disease due to loss of dopamine Association of polyarteritis nodosa with HBsAg. Case study of 2 woman with gonorrhea: probable PID shortly after menses Anorexic patient has more risk for osteoporosis due ta ioss of estrogen Left sided heart murmurs and abnormal heart sounds increase in expiration Right sided heart murmurs and abnormal heart sounds increase in ixspiration ‘Aortic stenosis: « systolic ejection murmur with radiation into carotids (S, and increases on expiration), + diminished pulse, + MCC is congenital bicuspid aortic valve, © MCC of microangiopathic hemolytic anemia with schistocytes, « MC valvular lesion associated with syncope and angina Description of an x-ray of emphysema: vertically oriented heart Tension pneumothorax, spontaneous pneumothorax, and atelectasis: + spontaneous~ subpleural blebs in tall, slender males is a common cause of spontaneous pneumothorax, pleural pressure same as atmospheric pressure, lung collapses, diaphragm rises, trachea shifts to side of collapse, decreased breath sounds, tympany to percussion, » tension pneumothorax tear in pleura acts like check valve, pleural pressure higher than atmospheric, lung compressed and not collapsed, mediastinal structures shift to opposite side, diaphragms low, decreased breath sounds, tympany to percussion, « atelectasis (collapse of alveoli)- decreased percussion, high diaphragm, increased tactile fremitus, decreased breath sounds, inspiratory lag, trachea shift to side of atelectasis Chronic alcohol abuse leads to congestive cardiomyopathy and cardiac failure: could be duc to thiamine deficiency or direct toxic effect on the heart ‘Type Hl pneumocytes are the repair cells of the lung in pneumonia and ARDS: lamellar bodies (surfactant) are cytoplasmic markers of the cell Drug addict with fever and pansystolic murmur increasing on inspiration: wicuspid regurgitation due to S. aureus infective endocarditis ‘Type 2 diabetic with no diet control: Heb Alc is high increased A-P diameter, » depressed diaphragms, * 2 Note: This material is copyrighted. All rights reserved. Edward Goljan, M.D, 2002 4 § PHHTE q q e444 8a qaqa ae”d q 4 gad Polycystic ovarian syndrome: + |7-ketostervids ‘ cevere are elevated, © LH.> FSH (DHEA and androstenedione), testosterone, Benzene: causes aplastic anemia and acure leukemia ‘Testicular torsion: testis raised on affected side and loss of cremasteric reflex ‘cal case of ulcerative colitis in young man; bloody diarrhea ical case of an elderly man with prostate cancer: © remember osteoblastic metas tower lumbar vertebra, « if the history is only one. : in at night, itis prostate hyperplasia tory is only one of winery retention and problems with geting up 21-Hydroxylase deficiency: * salt loser, » increased 17-KS and decreased 17-1 increased ACTH, « female pseudohermaphroditism a Ey Aenea LI- Hydroxytase deficiency: + hypertension— increased deoxycorticosterone, + increased 17-KS increased 17-hydrexyeorticaids (1 -deoxycortisol), « increased ACTH, » deoreased serum cortical, * female pseudohermaphroditism — 17- Hydroxylase deficiency: * hypertension increased mineralocorticoids, « decreased 17-KS, « decreased 17-hydroxycorticoids, * increased ACTH, © decreased serum cortisol, + Teale hypogonadism, « male pseudohermaphrodite 3 Cardiac tamponade occurs 3-7 days after an AMI Pregnant patient accidentally swallowed I will probably devel Duchenne's muscular dystrophy: SXR diciarwii sone at erect pestgrian PGE, synthesized in the placenta keeps the ductus arteriosus open during pregnancy In 2 patient with acute nephritis, what lab test should be ordered: ASO titers to R/O post- streptococcal glomerulonephrits Renal casts and their disorders: « RBC casts~ nephritic syndrome, « WBC casts cute pyelonephritis and drug induced tubulointerstitia! nephritis, renal tubular casts— acute tubular necrosis, # fatty casts~ nephrotic syndrome, * waxy casts— chronic renal failure Teratomas: « germ cell origin, « located in midline testes/ovaries, anterior mediastinum, pineal Glandular metaplasia of distal esopbagus: Barrett's esophagus in GERD Kwashiorkor: « has decreased oncoric pressure due to decreased protein intake but adequate number of calorics, + defects in cellular immunity, « anemia, « ascites, « fatty liver from decreased apoproteins Marasmus: « total calorie deprivation, + broomstick extremities . Mycoplasma pneumoniae is the MC organism for atypical pneumonia in adults ‘Neutrophils phagocytose monosodium urate crystals in gout Pancreas biopsy in cystic fibrosis patient: atrophy is the growth alteration Epidural hematoma: ruptured middle meningeal artery Rapidly progressive dementia with neurofibrillary tangles: Creutzfeldt Jacob Defective synthesis of type I collagen in Eblers-Danlos syndrome ‘Auer rods: « only seen in acute myelogenous leukemia (includes progranulocytic), + not seen in any chronic leukemia (includes CML) or monocytic leakemie Case of extravascular hemolytic anemia. Hyperseusitivity is equivalent to: EHA is a type Tl reaction, 50 look for another type Il reaction-¢.g., Goodpasture's, Graves, myasthenia Primary hyperparathyroidism electrolytes: « hypercalcemia, « hypophosphatemia, « normal AG metabolic acidosis Case of hemolytic uremic syndrome: + schistocytes, « D-dimers (fibrin strands with inks benveea them), « thrombocytopenia platelets consumed, « increased bleeding time, * normal PT and PTT not DIC Common complication of severe hemophilia A: hemarthroses Kiinefelter syndrome: « 47 XXY, + low testosterone, « high FSH and LH, « high estrogen Macrocytic anemia with neurologic symptoms: cobalamin deficiency (Bi) and 30 Note: This material is copyrighted, All rights reserved. Edward Goljan, M.D. 2002 4H 449 F499 Gag 99°99 9944 | q "4 Miroteloms: «assis expomme(hip-yd, roofer >20 yr) «nosing eaonhip Temporal arteritis: + granulomatous giant ce{l oun oveeent tema = Ssiris, * mereased sed rate, « jaw pain when Proximal dissecting aortic aneurysm: « aortic regurgitation murmur, « wi ineeaguha, openaetst Ineiengee ete ation murmur, « widening of sori knob, « Hypophysectomy: zona fasciculata undergoes atophy due to loss of ACTH Case of emphysema and pathology: « destruction of elastic pat < cenuilobular involves respiratory bronchioles, « panscinar involves we imine Meee es duct, and lveoti, + paraseptal involves the alveolar duct and tiveoli~ commen ca of preumothorax Down syndrome with vomiting and maternal polyhydramnios: duodenal atresia Down syndrome with constipation at birth: Hirschaprung’s denace ‘High risk of colon cancer: multiple polyps Elevated alpha fetoprotein in pregnant woman: do an amniocentesis to rule ‘out open neural tube fect Decreased alpha fetoprotein in preguant woman: Down syndrome Complications of prostate hyperplasia: « infection, » urinary retention, « it does not piers. cancer High serum TSH: primary (not secondary) hypothyroidism Thyromegaly, normal TSH and T,, increased antimicrosomal and thyroglobulin antibodies: early Hashimoto's thyroiditis Osteogenesis imperfecta: « deficiency of type I collagen, « blue sclera~ due to visualization of choroidal veins MCC of a fatty liver and cirrhosis: alcchol abuse ‘Case of mass in fallopian tube ina Patient with an increase in B-RCG: ectopic Bacterial meningitis: « increased opening pressure, + high protein, * low glucose, « increased neutrophils Brain tamor in the third ventricle: could be colloid cyst or choroid ptoxus papilloma Patient deterioration after CNS trauma; hemiation Kartagener's syndrome: absent dynein arm in cilia Rinne and Weber test are like if you have cerumen in your ear: « Rinne lateralizes to affected car and Webers has bone conduction > air conduction, « signs of conductive hearing loss Injection of I liter of saline, start seeing diuresis: due to an increase in auial natriuretic factor After head trauma, the arine esmolality is 700 mOsmukg: « inappropriate ADH syndrome, «not diabetes insipidus (low urine Osm) Primary hypoparathyroidism: primary due to previous surgery or autoimnume destruction 1,25 dihydroxycholecalciferol: « active form of vitamin D, « requires PTH for its synthesis 1 a- hydroxylase Graves disease: + type II hypersensitivity: IgG antibody against TSH receptor, « unique to Graves: exophthalmos, pretibial myxedema, * TSH low and T, high, « Rx with B-blockers to inhibit adrenergic effects and propylthiouraci to inhibit enzyme synthesis Differences between primary/secondary hypothyroidism in relation to T, and TSH: » both have low T, © primary has high TSH, « secondary has low TSH Woman who was vomiting: « hypokalemia, metabotic allalosis normal AG metabolic acidosis, « normal serum sodium (isotonic loss), « hypokalemia A patient in the hospital accidentally gets infused with saline contaminated with £. coli and develops shock, 2 days later there is bleeding from all puncture sites, what will be the findings: « DIC with decreased fibrinogen and platelets, « increased D-dimers 31 Note: This material is copyrighted. All rights reserved. Edward Goljan, M.D. 2002 @ a9 #€4 sang a4 a4 A.16 year old football player suddenly dies during a Ae ee pearl Ties ienmateaar apeaaeper gee eee ‘A person develops congestive cardiac failure and decreased GFR. The renal cobale cals hydrople change. What is the cause of the bydropic change?: damage to the Na-K om Graph showing collagen formation after an AMI: graph that stars and increases afer 19-14 da An AlDs patieat develops diarrhea with an acid-fast organism. What is the most likely ‘cause?: Cryptosporidium © A patient with liver cirrhosis and ascites develops 5 scent 5 thie saopt Ukaly orjaaisn?: Bvcoll Sapiens Peritenitis, vehot is A patient is being treated for leukemia with anti-cancer drugs. hiring hoes RSE id of Hiney: sine A person has a crescende-decrescendo murmur heard that radiates to the carotids, what is the defect”: aortic Sg ene econ tae Which type of hepatitis is not affected by alcohol?: HAV 46 XX female is born with male genitalia and a vagina that ends in a blind pouch, bas hypertension and hypernatremia?: « adrenal hyperplasia, « 11 hydroxylase deficiency Baby is bora with paffy face, sluggish and diminished deep tendon reflexes?: cretin Ghypothyroidism) : A systolic murmur is heard io the mitral area. It increases in intensity by 1 grade after a pause. What murniur is it?: mitral regurgitation Girl comes in with corneal rings, hepatic and brain changes. What mineral defect?: copper in Wilson's disease Patient comes in with leukemia. Over the past year she has developed a decreased hematocrit with microcytic bypachromle anemia: anemia of chronic disease Pregnant woman has polyhydramnios. What does the fetus have?: anencephaly ‘Tumor with loss of pS3 suppressor gene. Loss of regulation at what point?: Gl to S phase Boy with pica for paint lias anemia with basophilic stippling. What will be the finding in this patient?: increased fice erythrocyte protoporphyrin in Pb poisoning 90 yr old lady suddenly dies. On antopsy the cerebral veatricles are enlarged. What is the most likely cause?: cerebral atrophy : Previously healthy 12 yr old girl develops aplastic anemia. Canse?: parvovirus Different diagrams of wound healing and must identify which one is correct. Pick the one with the basal layer of the epidersois joining with the one on the other side. Patient with small-medium vessel disease is HBsAg positive. Diagnosis?: polyarteritis nodosa Graph with increased basophils, neutrophils and monocytes. What is the cause?: GM-CSF Patient bas a stab wound to chest and there is lung collapse. What is the pressure inside the plearal cavity?: same as atmospheric pressure Scenario of 2 man who bas had a stroke and died. Autopsy shows brain with hemorrhage. What is the eause?: stroke involving lenticulostriate branches of the MCA due to hypertension Inheritance patterns for genetic disease: see High Yield pathology Man working with rocks develops pulmonary symptoms: silicosis Know Rh and ABO hemolytic disease of newborn: sce blood bank notes Budd-Chiari syndrome (hepatic vein thrombosis): see hepatobiliary notes Physical diaguosis of lang conditions: + tactile fremitus- increased in consolidations and decreased in pneumothorax, + egophony- ¢ sounds like a, sign of consolidation, + percussion decreased in consolidation, pleural effusion, tympanitic in pneumothorax ‘Scar tissue in brain; called gliosis and due to astrocyte proliferation ; rae most affected by decreased oxygen: renal medulla (particularly staight portion of proximal tubule) followed by neurons in the brain Hypertensive brain injury: intracerebral hemorthage Patient with acute pyelonephritis: WBC casts 32 Note: a4 a4 4 a4 VETTVadedg a4 a “Tuls material is copyrighted. All rights reserved. Edwara Goljen, M.D, 2002 Complications of anorexia nervosa (marathon runner): « decreased GnRH, * osteoporosis 3}: * secondary amenorthea due to Patient with bilateral hydronephrosis: urethral obstruction murmem Increnied 26 ether; Alfosterone due wy deceit caidian uput and = decreased metabolism by liver Organ secondarily affected in cirrhosis of liver: sp ; Pancreatitis: pain referred tothe back, since itis reuoperaregt ou © POTal hypertension Amniotic fluid embolism: causes DIC Serum haptoglobin: decreased in intravascular hemolytic anemia Robe ef anemia in chronic lymphocytic lenkema:aoimmune hemolytic anemia Neonatal respiratory distress syndrome: no surfactan, Findings 30 min after acute myocardial Infarction: no changes Phosphorus increased in chronic renal failure MC TS site én reactivation: + upper lobe of lung, «high O, tension Pancoast tumor causes Horner syndrome: destucton of superior cervical ganglion in posterior Gp 120 attachment of HIV virus to CD, molecule Pedigrees: all types + mitochondrial DNA Causes of children getting a disease when parents don't have any evidence of it: incomplete penetrance im an autosomal dominant disease Patent mith a history of & bone marrow transplant and who is om Cyclosporin, has neutropenia and devclaps esophagitis: « picure of inwanucleasincusion= ifa single meleror Howl ov, pick CMY, ifmulninucleated cell with inranuclear inclusions pick Hones Enzymes decreased in lead poisoning: ferrochelatase and ALA dehyarece pauent witha bistory of bypertension develops an intracranial hemorrhage. What part ofthe Drain is affected?: basal ganglia (putamen isthe most coramon area) Huntington's disease: « caudate atephy, « dementia, « odd movements, © autosomal dominane, « trinucleotide repeat disorder Firefighter with a history of dyspnea 6 hours following a fire. Eistologic section of alveoli with hyaline membranes: ARDS Patient with septic arthritis. What is most likely organism”: Neisseria Alcoholic with a histary of depression and hypoglycemic episodes. Low blood glucose and no: ‘C-peptide: patient has been taking insulin Cause of acute renal failure ia 2 patient in a nursing home who hasn't eaten or drank anything for a while. Patient is dehydrated, has a history of a stroke 6 months ago, and has increased BUN and creatinine from previous values. He has pyuria but no hematuria, He has not urinated in 12 hours. After catheterization, 100m. of very dark yellow urine is removed, ‘What is it?: probably prostatic hyperplasia causing urinary retention and infection Cause of pneumonia in cystic fibrosis: Pseudomonas aeruginosa An alcoholic has 2 Klebsiella infection involving the upper lobe of the right lung, what kind of necrosis is it: liquefactive necrosis Granular IgG and C; deposits are found in the glomerular basement membrane: type memibranoprolifcrative glomerulonephvitis (dense deposit GN) What part of the kidney is most involved in diabetes: glomerulus 7 Tall, skinny 17 year old basketball player with long arms and legs and hyperflexible joints, what is the boy likely to develop: Marfan's with danger of dissecting aortic aneurysm A 6 year old boy ingests rat poisoning, along with supportive therapy, what should be given: IM vitamin K injection ; Prenatal vitamins: «iron: women only have 400 mg of iron stores and lose 500 mg in pregnancy, * folate: only 3—4 mth supply in liver, « By, is not necessary unless a woman is a pure vegan 33 ee ee | 4 V994ad 4q 994 a4 Most cost effective way of preventing hepatic coma in cirrhotics: « decreases ammonia load to liver where urea cycle is dysfunctional Vitamin deficiency associated with depression; «nisin deficiency due to typtcph : fophan synthesizes serotonin, which is decreased in depression deficiency, Vitamin deficiency associated with a tea and toast diet in elde Vie rly patieat with bleeding gums: Patient being treated for TB develops » microcytic anemia: sideroblastic deficiency related to INH Rx — blastic anemia from B6 Vitamin D deficiency in 2 patient on phenytoin: increased metabolism by the cytochrome sysiem Pare vegan is breast feeding baby. Baby develops anemia: By, dsfiiency Chile with milk allergies develops macrocyic anemia: faking gous mule which is deficient in ic acid After am earthquake in southern California many develop pulmonary symptoms most likely casual organism: Coccidioides oe eas reduce protein intake, « ‘Spherule with endospores! Cause of anemia in a patient with nodular sclerosing type of lymphoma?: anemia of chronic disease Folic acid must be taken before conception to prevent anencephaly in subsequent pregnancy ‘Woman with incomplete mole. Genotype?: tiploid 69 OY eee Patient insensitive to burning of hands. Upper extremity muscle weakness: syringomyelia Fuzzy balls in apical lung cavities in a patient with massive hemoptysis: Aspergillus Solitary coin lesion in the lungs of a patient from Mississippi: histoplasmosis Alveolar macrophage with yeasts Patient with prosthetic heart valve has schistocytes im peripheral blood: prosthetic valve is malfunctioning causing hemolytic anemia MCC of bladder diverticula: prostatic hyperplasia Prevent keloid formation: intralesional injection of corticosteroids : ‘Mother and father blind but no family history of blindness, one child already blind, chance other child will be blind: » patients probably have homozygous AR disease and parents were asymptomatic carriers, « all children will have the disease (100%) ‘Vitamin deficiency in pheochromocytoma: vitamin C- catalyzes dopamine to norepinephrine Phases of acute inflammation: begins with transient vasoconstriction before vasodilatation from histamine release Yellow mass extending up inferior vena cava: renal adenocarcinoma invading renal vein with extension up IVC Granaloma in AIDS patient: would have macrophages but they would not be activated to lal phagocytosed organisms since CD, T helper cells are gone —>—~Canse of increased serum cortisol and thyroxine in pregnancy: + increase in synthesis of their respective binding proteins by estrogen stimulation of liver, « TSH and ACTH are both normal, « free hormone levels arc normal. Mother. with polyhydramnios and fetus with anal atresia: VATER syndrome~ vertebral defects, anal atresia, tracheoesophageal fistula, radial (absent) and renal abnormalities Patient with deep venous thrombosis only given warfarin (not ciara) cpa os cine opagation of venous clot: heterozygote protein C deficiency, © short life of protein aes ibe patient hypercoagulabie in 6-8 hrs, since factors V and VIII are increased due to 4 ‘Note: This material is copyrighted. All rights reserved. Edward Gotjan, M.D. 2002 complete absence of protein C in the blood, «patient could also have developed hemorrhagic skin necrosis, 7 Patieat develops oliguria after an intravenous pyelogram: » developed — tubular necrosis, « dangerous to order an IVP in multiple photo Rheumatoid factor: igM antibody against gG h* ¥SlOmm and diabetes melas Child whose father has a history of osteogenic sarcoma dev. . ph sn ee Sen ot ae osteogenic sarcoma and breast cancer are other cancers associated with this suppressor gene, » likely that this was an autosomal dominant inheritance partcrn, so one of the Rb suppressor pees so Teaenaaasiiien birth, * ee a mother's side are all normal, thea the child's eye lem wo. sporadic jon (rrutation uppressor chromosomes have to be inactivated. pHa eee poneeP ae w Weight lifter develops numbness and tingling in the arm while sleeping: thoracic outlet syndrome with compression of subclavian artery and brachial plexus by hypertrophied anterioy scalenus muscles = Male child with chronic infections with a granulomatons reaction: » chronic grenulomatous disease of childhood, « XR inheritance, * absent NADH ‘oxidase in neutrophils and monocytes therefore cannot convert molecular oxygen into superoxide free radicals (absent respiratory burst) and cannot Kill bacteria, especially Staphylococcus aureus, © without superoxide, peroxide cannot be produced by superoxide dismutase, * without peroxidase, myeloperoxidase cammot combine peroxide with chloride to produce bleach. ‘+ Chromosome 21 codes for an Alzheimer precursor protein (APP), part of which is amylokd-B (A-B) protein: + toxic to neurons, association with Down syndrome (3 fimctional chromosome 21s) MCC of Alzheimer's in patient's under 40. 7 — Apolipoprotein gene E, allele e4 located on chromosome 19: « produces a product that increases the neurotoxicity of the Af protein in Alzheimer's disease, + cause of familial late-onset type Abnormality on chromosome 14 in Alzheimer's disease: synthesizes a Tau microubule- associated protein located in neurofibrillary tangles * Hamartoma: + overgrowth of ssue normally present in the organ, * bronchial hamartoma: increase in cartilage, * Peutz Jeghers polyp, * hyperplastic polyp in the sigmoid colon, + angiomyolipoma of kidney: seen in tuberous sclerosis : = Heterotopic rest (choristoma): * normal tissue in a place it is not normally present, « pancreatic ‘tissue in wall of stomach, « gastric mucosa in a Meckel's diverticulum: 2 Patient with restrictive lung disease, Raynaud's phenomenon, renal disease: © progressive systemic sclerosis (scleroderma), + esophageal dysmotility is another major problem leading to replacement of smooth muscle by collagen, « dysphagia for solids and liquids * AAT deficiency: * AR disease, « AAT is an anti-clastase that destroys clastase (protease) released by neutrophils, © children with certain phenotypes develop cirthosis: PAS + AAT in the hepatocytes (defect in secretion of AAT out of hepatocytes), » young adults with certain phenotypes have complete deficiency of AAT and develop panacinar emphysema (entire respiratory unit destroyed) in lower lobes (absent cil globin peak on a serum protein electrophoresis) + First step im management of tension pneumothorax: « positive pressures in pleural cavity push the lungs and mediastinum in opposite direction, hence compromising respiration and venous rerum to the heart, « insert needle into second intercostal space (anterior chest) to release positive pressure buildup in the pleural cavity _ Graves disease: IgG antibody directed against the TSH receptor: type Il hypersensitivity reaction Newborn with tetany and small ears: ¢ DiGeorge syndrome, » 3rd and 4th pharyngeal pouches do not develop (absent thymus and thymic shadow and absent parathyroids), * expect hypocalcemia a4 94 35 Note: This material is copyrighted. All rights reserved. Edward Goljan, M.D, 2002 a4 a4 NSAIDs: « prolongs bleeding time: inhibiting cyclooxygenase : platelets cannot aggregate, * normal platelet count and Tema ca pt Det Nine month old black child has dactylitis: « sickle cell crisis, « sickle cell di a eit hoe mutation) where valine replaces glutamic acid im the 6th position of the B Pernicious anemia: » autoimmune destruction of parietal cells, » ‘ deficiency) and acid production, eachlochydria eases epee ee Sexe Bia femophilia A: « SXR, edeficient VII: coagulant Sian to activate factor X ongulant, VITK complexes with platelet factor 3, DXa, and NSAIDs (e.g., indomethacin): « decrease PGE;, which normally maintains the Tae Sechelt ita dooms prcoonsea igen cones ol pepeicuioos, epee ‘Women with excessive weight loss (e.g. marathon runner, aerobics instructor, anorexia nervosa): decrease secretion of GnRH from hypothalamus leading to decrease in gonadotropins—> secondary amenorrhea» osteoporosis (lack of estrogen) Woman in 40's with malignant ascites and large ovarian mass: probably a mucinous cystadenocarcinoma: very large ovarian tumor, often associated with pscudomyxoma peritonei, if it was bilateral, then a serous cystadcnocarcinoma would be most likely ° Posterior acute myocardial infarction, right heart enlarges in 1 week, pulmonary capillary wedge pressure is decreased: « probable right ventricular infarction with pure right heart failure, « potentially could be a pulmonary embolus producing right heart strain Gross (G) and Microscopic (M) changes in an acute myocardial infarction (AMI): © 0-4hrs— no G/M change «4-12 hrs~e no G change, « M shows coagulation necrosis after 6 hrs . 12-24 brs— « G shows early pallor, » M shows more advanced coagulation necrosis © 1-3 d-« G — definite pallor, » M — neutrophilic infiltrate coming in from the perimeter of the infarct + advanced coagulation necrosis . 347 d- «© period ‘of maximal softness, « time for ruptures, ¢ macrophages move in from the perimeter to remove dead tissue © 7-10 d-« Gis the same as 3-7 d, + M shows collagen deposition yromegaly, normal thyroxine and TSB, presence of antimicrosomal and antithyroglobalin antibodies: early stage of Hashimoto's thyroiditis Patient develops dementia after instrumental involvement in the brain: CJ disease due to rions Baby with syndactyly: defect in apoptosis, which normally would have caused regression of thase tissues between the fingers and/or toes ; Bipolar patient develops polyuria: probably on lithium which produces nephrogenic diabetes insipidus Hematopoiesis in last month of fetus: occurs in bone marrow ; Baby chokes with every feeding and develops pneumonia: + TE fistula, blind proximal aehagus, = distal esophagus arises fom wachea: stomach distended, ¢ mother with polyhydramnios . Diethylstilbestrol: interferes with development of Mullerian structures | ; Patient with a central line develops hemiparesis: + central line is inseried either into the subelavian vein or the internal jugular vein, « clot develops in the central line ‘and embolizes to right syjum and enters systemic circulation through an atrial septal defect - Hyponatremia: could be due to an A. excess of free water Caaporopriate ADH on reabsorption of proportionately more water than salt (edema states like right ae fe or C. loss of proportionately more salt than water a diuretics, 21-hydroxy| 36 tea Note: This material is copyrighted. All rights reserved. Edward Goljan, M.D. 2002 deficiency, Addison's dseate with lots of mineralocorcoids, REA by rescricting water and salt iatake and using diuretics, Re Ge cere % Sex differentiation: « Y chromosome determines genetic sex: deaseeyre nal saline difereatanon of germinal sue into ovaries: wolfian duct suenice dere ean Of chromosome causes germinal tise to differentiate into teste: mailleran hibreny neers synthesized» apoptosis of millerian tissue in the male fetus, « fetal tatoree ge epididymis, seminal vesicles, vas deferens, « fetal il stcrone (DHT) develop prove (underscores why DHT is responsible for prostate hyperplasa/cancerandot estos ers male genitalia: fusion of the labia —+ scrotum, extension of clitons -» penis, © Sorcohecen converts testosterone into DHT ee True hermaphroditism: + patient has both male/female gonads: majority are 45 XX enone © ovary and testis, or votes, « * Psendohermaphrodite: + patient whose phenotype (appearance) is not in agreement wi genotype (tue gonadal sex), * male pseudohermaphrodite: genotypic male (XY a, Phenotype appears female (female genitalia), eg, testicular feminization, * femile Pseusiohermaphrodite: genotypic female (XX with ovaries), phenotypically resembles a male. eg. virilization in adrenogenital syndrome ” ‘7 — S-a-reductase deficiency: + male pseudohermaphrodite; not present in females, » absence of DHT effect on male fetus: testes located in inguinal canals, absence of prostate gland: no DHT effect, absence of ali millerian structures: no tubes/uterusicervis/uppet one-third of vagina since millerian inhibitory factor is present, extemal genitalia female due to absence of DHT effect blind vaginal Pouch, vagina partly milllerian in origin and partly of urogenital sinus origin, * presence of testosterone effect including: epididymis, seminal vesicles, vas deferens * Testicular feminization: + SXR disease: MCC of male pseudohermaphroditism, * deficieacy of androgen receptors: DHT and testosterone are present but cannot function without a receptor, « no milllerian structures; millerian inhibitory factor is present, « no male accessory structures: no testosterone effect, absent epididymis/seminal vesicles/vas deferens/prostate gland, * extemal genitalia remain female: no DHT effect, vagina ends as a blind pouch, « testicles located in inguinal canal or abdominal cavity; surgically removed owing to a risk for seminoma, « estrogen unopposed, since estrogen receptors are normal: secondary female characteristics are well developed, © patient 1s reared as a femaic, « laboratory findings: normal testosterone/DHT, increased LH: LH does not* respond to the negative feedback of testosterone * Smoker with flank pain and mass, hematuria: « renal adenocarcinoma, « derives from proximal renal tubular epithelium, « smoking MCC = Family tree with AD inheritance pattern showing disease occurring at am earlier age in affected members: « trinucleotide repeat disorder: disease gets worse in future generations due to addition of trinucleotides (c.g, CAG), © ¢.g., Huntington's disease an AD movement disorder associated with dementia would occur at an earlier age, + ¢.g., female carriers of fragile X (SXR disease) will develop symptoms of mental retardation (makes it look like an SXD disorder). 2 Stillborn of diabetic mother in poor glycemic control: likely see hyperplasia/hypertrophy of B- islet cells in the pancreas as a fetal response to maternal hyperglycemia Cirrhosis of liver: nodules are regenerative nodules composed of hepatocytes with no cord- sinusoid-cord pattem = Cirrhosis of liver: + antibody test that would be helpful in determining the cause would be ~ antibodies against HCV, antigen that would be useful is HBsAg Newborn has cataracts and vomiting: © probably an inborn error of metabolism (€.g, galactosemia or hereditary fructose intolerance), * order a urine for reducing substances: detects fructose and galactose water, Rx B by 37 Note: This material is copyrighted. All rights reserved. Edward Goljan, M.D. 2002 9 9 @ m TV infusion of thiamine (B1) in an alcoholic: thiamine is a cofactor dehyt a-ketogiutarate dehydrogenase, and a-ketoacid dehydrogenase, « To NADH, which, in tum, generated 6 ATPs * Teactions produce Electrolyte abuormalities in diabetic ketoacidosis: increased ani . increase in AcAc and B-OHB, hyponatremia: dilutional effect of water moving ea ee ee by osmosis, Ippertaenia ansecular kit of Kou of als moves ine te cel eee ae patient is deficient in K+ duc to osmotic diuresis, loss of excessive amount of Na” and Ke in ‘osmotic diuresis of excessive amount of Na" and K" ia ure: Child with petechia, thrombocytopenia, normal PT and PTT, no schistocytes: thrombocytopenic purpura, type Il hypersensitivity Se eee aaa Vegetarian with microcytic anemia: ¢ probable iren deficien i cy, plant irom is fe Sonvered into ferrous tobe reabsorbed, «very small rik for iron deficiency * Fame and must be /oman with facial rash, arthralgia, positive VDRLL with negative FTA-ABS: system erythematosus, note the biologic false positive syphilis serology due to anti-cardiolipin, saad Baisd re lesions i AIDS patent: « mos often duct Kapos’s scam (HSV), «cou also angiomatosis due to Bartonella henselae: silver stains identify organism, also cause cat scratch discase “demon — Animal with AB alleles crossed with animal with AC alleles. What i receive an AA organ? 75% peta Sepia cs aA Cc A AA AC BC Classic ABG patterns: PaCO2 HCO; G3-45 mm Bg) (22-28 mEq/L) A 69 Fs B. 26 u c. 68 34 D. 2 14 E. 48 38 F. 24 2 Acute uncompensated respiratory acidosis: barbiturate overdose, CNS trauma, ARDS, paralyzed diaphragm or other muscles of respiration Metabolic acidosis with partial compensation: any’ cause of increased AG metabolic acidosis (lactate, AcAc, B-OHB, formate [methyl alcohol poisoning], oxalate [ethylene glycol poisoning), renal failure, salicylate), any cause of normal AG metabolic acidosis (diarrhea, renal tubular acidosis) Chronic respiratory acidosis with partial compensation: chronic bronchitis, bronchiectasis, lungs in cystic Aibrosis Mixed primary respiratory alkalosis and primary metabolic acidosis: salicylate intoxication, endotoxic shock Metabolic alkalosis with partial compensation: vomiting, loop/thiazide diuretic, mineralocorticoid excess (primary aldosteronism) Respiratory alkalosis with partial compensation: pulmonary embolus, diseases, anxiety, restrictive tung 38 Note: This material is copyrighted. All rights reserved, Edward Goljan, M.D, 2002 7 Classic electrolyte profiles: Serum Na” Serum K* Seager Serum BCO; (138-147 mEq/L) (@.5-5.0 mEq/L) A. ns 3p) ORNS meV) 2-28 mEWl) B. 130 3.0 30 6 e 130 55 8 c D. 128 58 14 a E. 140 3.0 4 1 Fr 150 2.0 107 a Patient A has inappropriate ADH syndrome~ note the diluti : ta fas inaperop syn note the dilutional effect on all the analytes, especially a could has metabolic alkalosis due to vomiting or diuretics (thiazides or loops) atient C has increased anion gap (G2 mEq/L ie canaeatiiena gap ¢ ) metabolic acidosis. hyperkalemia is due to Patient D bas Addison's disease or a type [V RTA due to mine id deficien these patients have a normal AG metabolic acidosis and hyperialemiy sor note that Patient E has a normal anion gap metabolic acidosis due to diarrhea or type I or type I RTA~ note the hypokalemia : Patient F has mineraiocorticoid excess (eg. primary aldosteronism) = Asian man with nasopharyngeal mass: nasopharyngeal carcinoma, « associated with EBV 7 Myeloperoxidase deficiency: « acquired or genetic, « absent azurophilic granules in neutrophils in peripheral blood, © respiratory burst is normal— can generate superoxide FRs, « cannot kill bacteria~ * microbicidal defect, * no MPO to produce bleach * — Congenital adhesion motecule (B,- integrins) defect: * failure of the umbilical cord to separate in newboras, « no adhesion of neutrophils to the endothelial cells, « no inflammatory cells in the umbilical sturmp MCV Hypersegmented neutrophils ‘Autoantibodies: against IF and arietal cells. ‘Achlorhydria ‘Serum gastrin levels Risk of stomach adenocarcmoma Plasma homocysteine Urine methylmalonic acid Neurological disease ‘Schilling's test = Differential features of the microcytic anemias: bolded areas represent key differential points Fedeficiency ACD a,f-Thal minor — Sideroblastic (Pb poison) Mcv Low Low Low Low Serum iron Low Low Normal High TBC High Low Normal ow % Saturation Low Low Normal High Serum ferritin Low High Normal High RDW High Normal Normal Normal RBC count Low Low High Low 39 Note: This material is copyrighted. All rights reserved. Edward Gollan, M.D, 2002 Fedeficiency ACD o, B-Thal minor Sideroblastic (Pb pot RBCFEP High High Normal a (Pb poison) [ Hb electro, Normal Normal ce-thal: normal Nena es f }-thal: Marrow iron Absent High Rasy THe ArandE " Miscellaneous Ferritin best Hgb clectro. Comme basophil gold standard stippling. oe [ FEP = free erythrocyte protoporphyrin. RDW ~ red blood celll distribution width “ a * _ Summary chart with tab differential for polycythemia: [ Condition RBC mass | Plasma volame | S20, rytbropele Polycytheria rubra vera Tncreased | Increased Normal —| Boe ‘Appropriate polycythemia | Increased | Normal Decreased | Increased COPD. cyanotic CHD Ectopic polycythemia Tnereased | Normal Normal | Tnereased renal disease, HCC Relative polycythemia: Normal Decreased [Normal ‘Normal’ [volume depletion ‘CHD = congenital heart disease, HCC = hepatocellular carcinoma = Oxygen saturation (S20;) findings in congenital heart disease: ‘Normal | Patient | Patient | Patient | Patient | Patient S20, A B Cc D E Right arium 75 75 80 75. 75 zo | Right venmricle | 75 80 80 75 |i 80 Pulmonary art_|~75 80 80 80 75 80 (Pulmonary vein |_95 95 95 95 95 95 [Leftvenrricle {95 95 95 95 20, 20, Aorta 95, 95. 9s | 95 80 80 ‘SaO, = oxygen saturation A= VSD, B = ASD, C= PDA, D = Tetralogy, E = transposition > — Summary of pulmonary function tests in obstructive and restrictive lung disease: Parameter Obstructive Restrictive Total lung capacity(TLC) Increased Decreased Residual volume (RV) Increased Decreased Tidal volume (TV) Decreased Decreased Vital capacity (VC) Decreased Decreased FEV ine Decreased Decreased FVC Decreased Decreased FEV jue/FVC Decreased Normal to increased FEV ye: = forced expiratory volume 1 second, FVC = forced vital capacity = _ Comparison of ulcerative Colitis (UC) and Crohn’s disease (CD): Characteristic | Ulcerative Colitis Crohn's Disease Extent ‘Mucosal and submucosal ‘| Transmaral 4 Location Primarily targets the rectum. May | Terminal ileam involved in 80%.Terminal extend up into left colon in cont- ileum alone (30%), ileum/colon (50%), colon , inuous fashion (no skip lesions) or —_ alone (20%). Anal involvement 75%- ! involve entire colon. Does not involve | fissures, fisnulas, abscesses. Involves other | other areas of GI tract. areas of GI tract (mouth to anus) A are Characteristic Ulcerative Colitis Crohn’s Disease Gross features Inflammatory polyps— pseudopolyps representing areas of inflamed residual mucosa. Friable, red mucosa bleeds easily when touched, Ulceration’ hemorrhage. No skip areas. Fat creeps around the serosa, wall and narrow lumen. Skip os oe Gmportant feature). Strictures, fistulas (very i ic) in areas other than anus. Deep linear ulcers, cobblestone pattem, aphthoid ulcers (smail mucosal ulcers, ‘origin of linear ] ulcers, early sign). , Microscopic | Active disease: mucosal inflamma- | Very di: a 2 an | ' features tion with crypt abscesses (neutro- | granulomas (60%), transemurat | phils). Dyspiasia/cancer may be inflammation with subserosal lymphocytic + present. Chronic discase: neutrophils | infiltration, replaced by lymphocytes! plasma i cells. Dysplasia/ cancer may be | present. Complications | More common than CD: 1. toxic More common than UC: |. fistula ! megacolon— hypotonic/distended formation, 2. obstruction, 3. disease in other; bowel, 2. sclerosing pericholangitis, | areas of Gi tract, 4. calcium oxalate renal 3. HLA B27 + ankylosing spondylitis! | stones uveitis, 4. pyoderma gangrenosum, 5. adenocarcinoma-~10%, greatest risks are pancolitis, early onset of UC, and duration of disease >10 years Clinical Left sided abdominal cramping (not | Right lower quadrant colicky pain obstruction), diarthea with blood/ mucus, rectal bleeding, tenestaus (obstruction in area of terminal ileum) with diarrhea, Bleeding if colon involvement. i : i Anti-HBe-IgM_ Ant-HBs “| Interpretation negative | negative positive negative negative | serologic gap positive | positive positive negative negative | acute infection or chronic if >6 mths negative _| negative negative positive Positive | recovered from HBV | negative _| negative negative negative | _positive | immunized ! positive | negative negative negative negative _| earliest phase of acute HBV | = Summary table for polyuria: CDI = central diabetes insipidus, NDI = nephrogenic diabetes insipidus i POsm post H,O0 UOsmpostH,O UOsm post ADH (vaso deprivation deprivation —_pressin) injection NORMAL 292 750 760 CDL 319 110 550 NDI 312 98 120 PSYCHOGENIC 288 760 780 4l 7 _ Summary table of thyroid disorders: Disorder ‘Serum T, Grave's Increased Factitious thyrotoxicosis Thyroiditis (acute, subacute) Primary hypothyroidism Increasea Vi \ Gashimoto) al Secondary hypothyroidism [Decreased | Decreased | Decreased | Decreased TB _ Chypopituitarismy/ypothal) L ' Increased TBG Increased | Decreased omar (c.g. increased estrogen) Nomat [Normal [Neral Decreased TBG. Decreased Incre: i i cased Nom (increased androgens) Nomnal | Normal a ‘= Summary of laboratory studies in Cushing's Syndrome: - Laboratory Test Pituitary Cushing’ ing” i Seram Cortisol Treesed asad ne® Peon Cashing’ Urine for free cortisol Increased Increased ineressed Low dose dexamethasone Cortisol not Cortisol not Cortisol not suppressed. suppressed suppressed High dose dexamethasome Cortisol suppressed Cortisol not Cortisol not suppressed Suppressed Plasma ACTH. “Normal” to Decreased Markedly Increased Increased @ — Non-caseating granuloma in an African-American with hypertension: sarcoidosis = Soft, tender lump in the inguinal area of a child: cryptorchid testis = Kidney of hypertension: nephrosclerosis— due to hyaline arterioloscterosis Woman with a3 cm mass removed from breast. 1 year later, the lung is involved. Mechanism of spread?: hematogenous 2 Increased intraocniar pressure and swelling of retina: glaucoma Woman with left flank pain, microscopic hematuria. CT sbows enlarged bladder and kidney. Site of obstruction?: bladder-urethra junction * * — Crateriform lesion on inner side of eye. Site of origin?: basal cell carcinoma— derives from basal cell layer ® — Parotid mass with and without atypia: » with atypia is a mucoepidermoid carcinoma, « without atypia is pleomorphic adenoma = Patient with AIDS has a space occupying lesion in the brain: toxoplasmosis HIGH YIELD NOTES ANATOMY ® Arteriogram of upper arm with 2 history of an absent radial pulse: damage to brachial anery- extension of the axillary artery and divides into the radial and ulnar artery Bypoglossal nerve in brain stem: exits in medulla at preolivary sulcus Cerebellum: Purkinje (rabies inclusions), mossy fibers in lower cerebellum Spinal spinothalamic tract and medial lemniscus: both go to thalamus ‘Types of junctions in lateral region of cells: # zona occludens (tight junctions), * zona adherens, « desmosome, « gap junction (nexus, passage of materials, dye passes from one cell to the next) Leaky junctions: present in proximal tubules of the kidneys Actin: present in muscle in iris but are not responsible for motility in sperm a4 a9ee 4 qq 42 ‘Note: This material is copyrighted. All rights reserved. Edward Goljan, M.D. 2002 944 14 4aq 94 944 signs (loss of lateral corticospinat tract), » senteiibratory sensation from a sien lets ee He disciminaton/posiional rain stem for localization of CN VII: patient Seeesin, side and buceinator and facial muscles on ight se ows PAYS Of ofbiculasis oes on right Brain stem for localization of CN VIN: « cerebellopontine angle NE sory ‘neural hearing loss, » located at Brain stem for localization of CN VI: history of problem with lateral gaze- loss of function of Solitary nucleus: receives general visceral afferents afferents fom CN VIL, IX, and X and projects thes te th none no ea Siliiciaiasin Neuroanatomy of brain slices for locations of structures involved in different diseases; « caudate nucleus— absent in Huntington's chorea, « lenticular nucleus~ degenerated in patient with Wilson's disease and is associated, « substantia nigra— see depigmentation in Parkinson's disease Hemiballismus in patient with Wilson's disease: degeneration of the subthalamic neste. Tabes dorsalis: dorsal column syndrome loss of tactile discrimination, loss of joint and vibratory sensation, paresthesias and pain (dorsal root irritation), astercognosis, hypo/areflexia (dorsal root deafferentation), urinary incontinence/eonstipation/impotence (dorsal root deafferentation) Romberg’s sign (standing patient unsteady with eyes closed but okay when open) CT of abdomen: kidney is bchind the pancreas CT of thorax: identify structure at the level of the sternal angle Mascte that attaches at iliac erest: « gluteus medius, which abducts and rotates the thigh « distal attachment is greater trochanter of femur Visual field defect with eraniopharyngioma or acromegaly: bitemporal hemianopsia- impinges on optic chiasm Outpouching of floor of diencephalon: cerebrum ‘Neurotransmitter of raphe ancleus: serotonin Location in a cystourethrogram of the urogenital diaphragm: © muscles are deep transverse perineal and sphincter urethra muscle, « in male, it is located just distal to prostate, « in female, probable at beginning of urethra when it exits the bladder Spinal cord site for loss of two point discrimination in left leg: fasciculus gracilis in dorsal columns covers two point discrimination for the ipsilaterai leg (closest to midline), « fasciculus cuneatus (lateral to cuneatus) covers two point discrimination for the ipsilateral arm Spinal cord site for loss of pain sensation in left leg: lateral spinothalamic tract on right side of cord (remember the cross-fibers) Patient with dyscalculia, finger agnosia, left/right confusion: + lesion in visual association cortex (angular gyrus) in left parietal lobe, + called Gerstmann's syndrome Papilledema: swelling of the optic nerve~ indicates an increase in intracranial pressure Omphalocele: » midgut loop fails to return to abdominal cavity, © shiny sac visible at base of umbilicus X-ray of duplicated ureter: congenital anomaly Horseshoe kidney: lower poles fuse, « kidney trapped behind inferior mesenteric artery Nerve cut producing sensory loss on medial side of leg when taking saphenous vein for CABG procedure: saphenous nerve, which is a branch of femoral nerve Note: This material is copyrighted. All rights reserved. Edward Goljan, M.D. 2002 a4 V99NGdg ag 4 a4 a% 9% 4 a4 CT of abdomen: structure that splenic vein empties into: « empties into portal vein neck of pancreas), « PV is composed of splenic vein and superior Gosterior to aon aes cade by foliage splenic vein ‘Desenteric vein (largest vessel), « ‘Ulnar nerve functions: * medial epicondyle injuries, * normally adc ‘of paimar and dorsal interosseus muscles, « adducts thumb sacra pense: us a Gerad to hemiated disc compressing root of plasma cell: sceretes antibodies, prominent rough endop! ek mithearakeal appearing nuclear chromatin endoplasmic rectum, cecenric muleus Characteristics of postganglionic peripheral nervous system fibers: « . Duodenal ulcer perforation and bleed, petolocend oer ‘bers: shor,» near end organ pa of knee joint in a patient with a positive posterior draw sign: identify posterior cruciate ligament Bicormuate uterus: failure of fusion of paramesonephric duct CN V: goes through superior orbital fissure ‘Melatonin: produced in the pineal gland Abdominal incision to avoid interrupting blood supply: midline Cleft palate: maxillary prominence fails to fuse with medial nasal prominence ‘Hematopoiesis at 28th week: bone marrow main site Schwann cel/oligodendrocyte: » Schwann cell makes myelin for PNS oligodendrocyte for CNS, « ‘Schwann cells develop from neural crest cells Location of neurohypophysis in the pituitary: neurohypophysis is posterior lobe where ADH is stored ‘Loss of pain and temperature on side of face: CN V lesion Derivation of adrenal medulla: « neural crest origin (S100 antigen Positive), © neuroblasts develop into ganglia, * know the layers of the adrenal gland from outside in from the cortex to the medulla— glomerulosa (aldosterone), fasciculata (cortisol), reticularis (sex hormones), medulla (catecholamines) Tibial nerve fauction: + plantar flexion of toes, + injury causes~ + loss of plantar flexion, « foot Gorsiflexed and everted (calcaneovalgus-cavus), * sensory loss on sole of foot Composition of aortic valve (also pulmonic valve): « lined by endothelium and have abundant Abroclastic tissue plus a dense collagenous core, « avascular, * MV and TV have a loose connective ‘issue core composed of dermatan sulfate which is increased in mitral/ricuspid valve prolapse (myxomatous degeneration) Break humerus, wrist drop; radial nerve injury Post-radical mastectomy—winged scapula: indicates injury to the long thoracic nerve; paralysis of the serrams anterior muscle Patient with 2 nosebleed and rhinorrhea: fracture of cribriform plate in ethmoid sinus Medial lougitadinal fasciculus demyelination in maltiple sclerosis: bilaicral internuclear ophthalmoplegia Parathyroid derivation: third and fourth pharyngeal pouches Aorde arch derivatives: « first- part of maxillary artery,» second stapedial and byoid artery, « third— common carotid artery and proximal portion of the internal carotid artery, « fourth aortic arch on the left and proximal portion of the right subclavian artery on the right, « sixth~ proximal portion of the pulmonary artery on the left and the ductus arteriosus Nerve that runs along the radial artery: median nerve Artery affected in femoral neck fracture: medial femoral circumflex artery- damage leads to aseptic necrosis of the femoral head EM of egg: where does sperm penetrate (zona pellucida) Note: This material is copyrighted. All rights reserved. Edward Goljan, M.D. 2002 o aq qq er 94 a9 Types of collagen: « [- * bone, + tendon, * skin, * greatest tensile strength, * Il. « ‘of wound repair, * replaced by type I via collagenase with Zn as Sen . Tet collagen membrane, ¢ X- epiphyseal plate ‘basement Wallerian degeneration: ¢ sprouts are remyelinated and reestablish continuity with the motor end plate of the mune Cirete of Willis: name arteries and mow how to identify them on an angiogram CT liver: hepatic vein drains into the inferior vena cava Eye closed cannot open and eye deviated down and out: oculomotor nerve palsy; eye dowa and inz trochlear nerve palsy Blunt injury to the back of the throat: potential injury to the injury to the cervical sympathetic ganglion ptosis and meiosis of the right eye: Vertical diplopia: cranial nerve IV palsy Patient with headache and physical findings of mydriasis In the right eye in association with mild lid lag, and deviation of the eye down and oat: an aneurysm compressing cranial nerve II. headache is the giveaway for aneurysm Patient with a recent history of bacterial meningitis has horizontal diplopia in the left eye, which is worse on gaze to the left: cranial nerve VI palsy- lateral rectus weakness from VI nerve palsy Patient with bilateral lateral rectus muscle weakness: increase in intracranial pressure~ + classic sign, * papilledema usually present Paralysis of upward gaze in an infant: » hydrocephalus secondary to stenosis of the aqueduct of Sylvius, « this is called Parinaud’s syndrome Multiple ocular motor nerve disorders: diabetes mellitus- common cranial nerve palsies from osmotic damage to nerves Weakness of the quadriceps muscle and an absent knee jerk reflex: hemiated Ly-L. disk Fall on outstretched arm with pain in the middle and lateral portion of clavicle and upper extremity remains in abduction, extension, and internal rotatiou: © nerve injured is C;-Ce- Erb-Duchenne syndrome, or superior brachial plexus injury due to a clavicular fracture (most common fractures in newborns), ¢ Cy-T, are inferior brachial plexus injures or Klumpke’s syndrome Patient has paralysis of the ocalomotor nerve after a bead injury: uncal hemiation with compression of the Ilird nerve— ptosis of eye, mydriasis Numbness of the thenar aspect of the hand: median nerve (carpal nmnel) ‘Wrist bone with greatest incidence of aseptic mecrosis: scaphoid bone Fetal circulation: ductus venosus and umbilical vein have the highest oxygea content EM of alveolus with macrophage, type IT pnenmocytes (lamellar bodies [surfactant]) EM of small bowel: look for microvilli on the surface ; Histologic section of seminiferous tubule: Sertoli cell synthesizes sex hormone binding globulin and also synthesizes inhibin, which has a negative feedback with FSH Bands in skeletal muscle: A band has myosin ATPase: contracts Respiratory bronchiole: « last airway structure with cilia, © respiratory unit where gas exchange occurs is the respiratory bronchiole, alveolar duct, and alveoli J : Terminal bronchioles: ¢ cilia but no goblet cells, « site of obstruction in asthma, cystic fibrosis, and chronic bronchitis, ¢ site where turbulent air flow becomes laminar due to parallel branching of the airways ; 45 ‘Note: This material is copyrighted. All rights reserved. Edward Gollan, MD, 2002 4 94444 q 4q 4 94 gage 99s C444 G Gg aqaaq9 Normal EM of 2 cell: © know all the normal organelles in Gua, Posttranslational modification of various compounds, * rough ead eie roe aPparatus~ symtiesis, + smooth endoplasmic reticulum» ste of eyachrome PASO mee ee Poet steroids, * site of y-giutamyl transferase im * synthesis of Voice hoarseness post thyroid sargery: injury to laryngeal nerve MRI of orbit: location of the superior oblique muscle MRI of abdomen: splenic artery is above the pancreas Chest x-ray: fluid in the cestophrenic sulcus in CHE X-ray with enlargement of the posterior heart: « enlarged left atrium in a stenosis, * most posteriorly located chamber in the heart, » see best with raed il MRI of the carotids with occlusion of the anterior cerebral artery: would effect the contralateral leg Schwannoma in jugular foramen: © weakness of palate/loss gag reflewlaryngeal paralysis (X). tePezius/stemocleidomastoid (XD, «loss taste sensation posterior thd of tongue (CX) ° Weber syndrome: « lesion of oculomotor nerve and UMN signs, © usually aseovisted with « midline, midbrain lesion Horner's syndrome: + diagram of vertebra and sympathetic trunk, « pick superior cervian sympathetic ganglion Papil light reflexes with eye diagrams of pupils: one set is an oculomotor nerve lesion with the light in the pupil in a down and out location Brainstem anterior view: find arca of oculomotor ‘nerve Frontal fobe lesion; affects personality Inferior quadrantanopia: defect in the superior fibers in the parietal lobe [Loss of sensation in the bands, history of burus without knowing it: syringomyelia in cervical cord and involvement of crossed spinothalamics G, transection of fasciculus gracilis: effects vibration and fine touch of lower extremity only CN TIL and UMN signs on opposite side: midline midbrain lesion Loss of pain and temperature and UMN signs on opposite side: mid pons lesion Horner’s syndrome with localization of lesion on a diagram: iateral medullary syndrome with associated cranial nerve palsy in the medulla and hypothalamus with associated temperance regulation problems Parkinsons disease: MPTP drug of abuse association~ derivative of meperidine ‘Cross-section of an embryo: identify neural crest tissue Blood production prior to birth: yolk stc-> liver» bone marrow Bochdalek hernia in posterolateral part of diaphragm on left: © present carly in life, « visceral contents extend into the chest cavity causing severe respiratory distress at birth, « parasternal diaphragmatic hemias extend through the foramen of Morgogni beneath the sternum and do not usually develop syerptoms until later in life Artery associated with foregut, midgut, hindgut: + foregut— mesenteric, » hindgut~ inferior mesenteric Damage to hearing in a rock and roll band player: injury to cochlea Small vs large bowel: plicae semilumaris go around entire circumference of small bowel and are interrupted in large bowel Argyil-Robertson papil: accommodates when patient follows finger moving towards the nose but does not react to direct light; neurosyphilis MRI of liver: hepatic vein- vessel emptying it he inferior er cva Pathogenesis of hypospadias: faulty closure of urethral fol Pathogenesis of eplapadias: defect genial tubercle assisted with exstrophy of bladder Feces draining from umbilicas in 4 day old: persistent umbilical (vitelline) sinus Urine draining from umbilicus in 4 day old: persistent urachal sinus midgut superior 46 a4 a4 eaq4 Femoral neck fracture: * bleeds into the capsule and may compromise medial femoral circumflex artery leading to avascular necrosis Embryology of miillerian duct, wolffian duct, urogenital sinus: « structures derived from the Mullerian duct include- * fallopian tubes, * uterus, * cervix, * upper 1/3rd of vagina, * in male fetuses, milllerian inhibitory factor (MIF) causes regression of the millerian duct by apoptosis, « in male fetuses, the paired wolffian (mesonephric ducts) form the epididymis, seminal vesicles, and Vas deferens * testosterone controls this, * dihydrotestosterone develops the external genitalia of the male fetus and the prostate gland, « in female fetuses, the wolffian duct normally regresses— inclusions may persist to form cysts (e.g., Gartner duct cyst in the vagina/cervix), * structures derived from urogenital sinus include * vestibule, + lower 2/3rds of the vagina ‘Anatomy/histology of ovary: « lined by coelomic epithelium surface derived ovarian tumors derive from this (€.g, serous cystadenoma), * outer cortex contains the follicles , * medulla contains mesenchymal tissue and steroid-producing hilar cells~ stromal thecosis of the hilar cells leads to hirsutism, « ovaries in a postmenopausal women should not be palpable— enlarged ovary is cancer until proven otherwise Oogonia at birth: + arrested in prophase I of meiosis and are called primary oocytes, + arrested cells are diploid (2n) and have 46 chromosomes Primary oocyte: « primary oocyte completes the first meiotic division within the mature follicle shortly before ovulation» secondary oocyte, + secondary oocyte is haploid (In) and contains 23 chromosomes, « it enters the second meiotic division but is arrested in metaphase II at the time of ovulation, * the secondary oocyte completes meiosis II during fertilization “Anatomy/bistotogy of cervix: « cervix = endocervix (mucous secreting columnar cells) + exocervix (squamous cells, begins at cervical os)}- squamocolummar junction (SCJ) is where the two epithelium's meet, « endocervical epithelium normally migrates down into the exocervix and replaces the normally squamous epithelium with mucus secreting colummar epithelium ° columnar cells exposed at the cervical os undergo squamous metaplasia (called the transformation zone), * the TZ is the area where squamous dysplasia end cancer develop Bilateral internuclear ophthalmoplegia: ¢ demyelination of medial longitudinal fasciculus, pathognomonic of multiple sclerosis, « when the patient is asked to look right, the right eye moves and has jerk nystagmus but the left eye is still looking straight abead, * when the patients asked to Took left, the left eye moves and has jerk nystagmus but the nght eye is stil looking straight ahead Rotator cuff tear: + common cause of shoulder pain, « components of the rotator cuff include the tendon insertions of- # supraspinams, * infraspinatus, « teres minor, * subscapularis muscles, « S/S of rotator cuff injuries~ pain/wealmess with active shoulder abduction ‘Shoulder dislocation: « majority are anteriorly dislocated, « often injure the axillary arvery/nerve Teanis elbow: « pain occurs in the area where the extensor muscle tendons insert near the lateral epicondyle, « common in— * raquette sports, * repetitive use of a haramer or screwériver Golfer's elbow: pain is located where the flexor muscle tendons insert near the medial epicondyle Ulnar verve compression: * the ulnar nerve may be entrapped in the following areas * transverse carpal ligament, * elbow (“funay bone area"), © there is pain and numbness of the ulnar aspect of the forearm and ring and litle finger plus weakness ofthe intrinsic muscles of the hand Reflex loss in C.-C; disc: biceps reflex associated with musculocutaneous nerve Cs-Cs Reflex loss in Cs—C, dise: supinator 3 Reflex loss in C.-C; disc: triceps reflex associated with radial nerve Ce-Ce DeQuervain’s disease: « chronic stenosing tenosynovitis of the first dorsal compartment of the wrist, « duc to overuse of the hands and wrist, « the first dorsal compartment contains the abductor pollicis longus (APL) and the extensor pollicis brevis (EPB)- excessive friction causes thickening 47 Note: This material is copyrighted. All rights reserved. Edward Goljan, M.D. 2002 of the tendon sheath and stenosis ofthe osseofibrous tunnel, « pain occurs on the radial aspect of the wrist and 1s aggravated by moving the thumb~ pain inthe region ofthe radial styloid process © — Ganglion cyst: » common cause of a bulge on the dorsum of the wrist when the wrist is flexed, © ganglion” (synovial) cyst is filled with mucinous material, © somet abil en ckenvanlts puichite bye ees Ea he, ihe ra > — Compartment syndromes: + increase of pressure in a confined space pressure reduces perfusion, which may lead to permanent ischemic contractures of the muscle (s) in that commpartment, « S/S of a compartment syndrome~ * pain, * paresthesias, * pallor, + paralysis, * pulsclessness, « causes of compartment syndromes- * fractures, * injuries to arteries/soft tissue, * prolonged limb compression (¢.g.. tight fitting cast), « Volkmann's ischemic contracture~ * it is a complication of a supracondylar fracture of the humerus, + there is injury to the brachial artery and median nerve, © brachial artery ischemia leads to increased pressure in the closed muscle compartments of the forearm with a subsequent decrease in venous and then arterial perfusion: this may lead to permanent ischemic contractures of the muscle % Carpal tunnel syndrome: + entrapment syndrome of the median nerve in the transverse carpal ligament of the wrist, * it may also be entrapped between the bellies of the pronator teres muscle along with the radial artery, * causes~ * RA/pregnancy, * overuse of the hands/wrist, © amyloidosis, * hypothyroidism, ¢ S/S—* pain, mumbness, or paresthesias in the thumb, index finger, second finger, third finger, and the radial side of the fourth finger, * thenar atrophy ("ape” hand appearance), + pain is reproduced by tapping over the median nerve = "Claw hand": « ulnar nerve palsy, « ulnar nerve functions— * adduction of the fingers due to innervation of the palmar and dorsal interosseus muscles, * adduction of the thumb (adductor pollicis muscle) = "Wrist drop”: « radial nerve palsy, « radial nerve fumctions- * extensor muscles of the wrist and digits, + wrist drop refers to a hand that is flexed at the wrist and cannot be extended, injury may bbe due to- * midshaft fractures of the humerus, * draping the arm over a park bench (called “Saturday night palsy") + "Waiter's tip deformity": « brachial plexus lesion involving C; and Cs upper trunk injury, © clinical findings in Erb-Duchenne's palsy~ * birth injury of the brachial plexus with damage to Cs-Ce nerves upper trunk injury, + loss of abduction of the arm from the shoulder, * inability to externally rotate the arm, ¢ inability to supinate the forearm, * absent biceps reflex. * asymmetric Moro reflex: no movement on the affected side & — Klumpke's paralysis: « birth injury of the brachial plexus with injury to the CC, and T,- lower trunk injury, paralysis of the hand, « Homer's syndrome Axillary nerve injary (C.-C): « fracture of the surgical neck of the humerus, * dislocation of the shoulder joint * usually anterior dislocation, * may also injury the axillary artery, » patient cannot abduct the arm to the horizontal position or hold the horizontal position when a downward force is applied to the amm- paralysis of deltoid muscle, * weakening of lateral rotation of the arm~ paralysis of teres minor muscle = Arteries, nerves, tendons cut with a deep laceration of the radial side of the wrist: « srtery— radial artery, « nerve~ median nerve: courses along the radial artery, « tendons * palmaris longus, + flexor carpi radialis 2 Arteries, nerves, tendons cut with a deep laceration of the ulnar side of the wrist: « arery- ulnar artery, * nerve— ulnar nerve, « tendon flexor carpi ulmaris ; & — Cause of “shin splints": repetitive loading of the anterior compartment muscles of the tibia * inflammation occurs at the musculotendinous insertions: these are often called "stress fractures" © — MC site for a compartment syndrome in the leg: anterior compartment of the tibia 48 Note: This material is copyrighted. All rights reserved. Edward Coljan, M.D. 2002 Mechanisms of low back pain: * MCC is spasm of the paraspinal anterior/posterior longitudinal ligaments, © nerve root irritation with nerve root compression, * diseases involving vertebral col myeloma, * osteoporosis with compression fractures = ees eee disc siete degeneration of fibrocartilage/nucleus pulposus + the nip isc material may hemiate posteriorly and compress the nerve root andor Spinal cord. Pain (sciatica) radiates from the low back to the buttocks, down the leg, below the knee, « §/S of ise herniation + leg pain is aggravated by straight leg raising * 2 — Herniation of (Les i somo loss~ » lateral and posterior calf, « plantar aspect of the foot, + reflex loss— Achilles’ reflex (tibial nerve L,-S,), « itm sat ee ed al ne Li-Si), « motor defict~ « loss of plantar flexion, «loss ‘7 Herniation of Lil dise: + sensory loss * dorsum ofthe foot, + webbed space between the great toe, « reflex loss none, » motor deficit loss of dorsiflexion of the big (great) toe due to weakness of the extensor hallueis longus * Herniation of Ly-Li disc: + sensory toss~ medial leg to the malleolus, « reflex loss~ knee jerk (femoral nerve Ly-L,), * motor deficit- * quadriceps weakness due to weakness of knee extension, + loss of dorsiflexion of the foot due to weakness of the tibialis anterior Cauda equina syndrome; » bowel or bladder dysfunction, « saddle area anesthesia Spondylolisthesis: + forward subluxation of one vertebral body on another, © MCC is spondylosis knee is acutely flexed-> foot is grasped and the leg is extemally rotated.» lore ve slowly extended while the other hand feels the posteromedial margin of the knee joint-> a slick along the posteromedial margin indicates a medial meniscus tear: femur passes over the tear = test. for a lateral menisous injury: same procedure as above except the leg is rotated intcmally and extended-+ a click is palpated along the posterolateral margin of the joint Location and function of the anterior cruciate ligament (ACL): ACL anaches the anterior part of the ubis to the fateral condyle of the femur— prevents anterior movement of the tibia in relation vo the femur Location and function of the posterior cruciate Ligament (PCL): PCL extends from the posterior part of the tibia to the medial condyle of the femur prevents posterior movement of the nbia in relation to the feraur Tests used to evaluate the cruciate ligaments in the knee: « anterior draw test to evaluate ACL— patient supine, hip flexed 45°, knee flexed 90°» examiner places hand on the posterior aspect of the Ubia—> anterior force is applied in neutral, external, internal direction» positive anterior draw test is when there is anterior displacement of the tibia, « posterior draw test to evaluate PCL— patient supine, hip flexed 45°, knee flexed 90°» examiner places hand on the anterior aspect of proximal tibia-> posterior force is applied in neutral, external, internal direction positive posterior draw test is when there is posterior displacement of the tibia Functions of the medial collateral ligament: supports the medial side of the knee joint- « attaches the medial epicondyle of the femur with the shaft of the tibia, * resists valgus and extemal rotational forces of the proximal tibia in relation to the distal femur Functions of the lateral collateral ligament: supports the lateral side of the knee joint « attaches the lateral epicondyle of the femur to the head of the fibula, * resists varus forces and rotational forces of the proximal fibula in relation to the distal femur S/S of a meniscus injury: © pain, + knee catches, locks, or gives way when walking, © swelling/popping of the nee Medial meniscus injury: + mechanism of injury~ * MC internal derangement of knee joint, « most _ commonly part of a valgus injury (¢.g., clipping in football), « structures damaged- + medial meniscus, * medial collateral ligament, * ACL, # positive McMurray's test~ click on posteromedial margin with the knee flexed, extemaily rotated, and slowly extended Lateral meniscus injury: * mechanism of injury— * varus injury, + injury to lateral collateral ligament, * positive McMurray’s test- click on posterolateral margin with the knee flexed, extemally rotated, and slowly extended 50 Note: This material is copyrighted. All rights reserved. Edward Gatjan, M.D. 2002 & ACL injury: + mechanism of injury + MC ligament injury, + to injury secondary to clipping or skiing, © positive anterior dra sign mm most commonly ina valgus 7 PCL injury: « mechanism of injury- + hyperextension of the lnce secondary pushing the tibia in a posterior direction, « positive posterior draw eat” ‘oancnanetie eres = Medial collateral ligament injury: tom with valgus injuries % Lateral collateral ligament injury: tom with varus injuries @ — MCC of am ankle sprain: sprain of lateral ankle ligamen sakeabs in Dashaeee, players, volleyball, ane foottall from inversion of plantar Axed fot > MC lateral ligament that is sprained: anterior talofibular ligament very stabilization of planar flexion in the foot Har Hgersat- very important ligament in Positions the knee joint i forced into that results in injury: « valgus positon~ angulation avay from the midline g, clipping injury in football, « varus position—angulation towards the midlng, medially originating force is applied to the knee MC nerve injured with clavicular fractares: ulnar nerve MC nerve injured in proximal humerus fractures: axillary nerve MC nerve injured in mid-shaft/distal third of humerus: radial nerve~ * nerve travels in the spiral groove, * wrist drop = MCC of pain in the elbow and inability to supinate the forearm in 1-4 yr old: subluxation of the head of the radius~ usually due to jerking 8fUie hand By an imipabent Sr abusing parent — 7 MC fracture associated with falling on the outstretched hand: * Colles fracture of the distal radius * MC fractare of the wrist, + radiologically, it produces a “dinner fork” deformity of the proximal radial fragment (displaced upward and backward), * second most common Face un osteoporosis in women * MC carpal bone fracture: scaphoid + pain in the anatomical snuff box located below the radial styloid process, * high incidence of aseptic necrosis 7 MC hip dislocations: = posterior usually due to a car accident with the flexed knee in an abducted position is forced into the dashboard, * limb is shortened, flexed, adducted, internally rotated at the hip, * danger of damage to sciatic nerve, » anterior « limb is shortened, » abducted, + extemally rotated at the hip, * neurovascular compromise of the femoral artery, vein, nerve, « aseptic necrosis of femoral head MC femoral fracture: + feroral neck fracture~ * it most commonly occurs ia the elderly male patient with osteoporosis, * groin or knee pain is present, + complications aseptic necrosis of femoral head due to damage to the medial femoral circumflex artery 7 — MC foot bone fractured after a fall from a height: calcaneus + MC fracture associated with ecchymoses of the mastoid, basilar skull fracture: + petrous portion of the temporal bone, « otorrhea (CSF fluid leaking out of the ear) may also occur). 7 MC fracture associated with rhinorrhea: » orbital fractures, © orbital fractures also produce raccoon eyes (periorbital hemorrhage and ophthalmoplegia (eye muscle entrapment). 7 Cross-section of spinal cord: + location of pain in right hip, © location for sympathetic preganglionic fibers & Shining tight in left eye causes pupil to constrict in left but not the right eye. When shining light in right eye, pupil constricts in the teft but not the right eye: right CN Ili dysfunction o-~-Pattene wittr spasticity-aud loss'of tactile dlsctitithatoW and vibratory sensation: subscilté combined degeneration in By, deficiency = Foreign body in alveolus: phagocytosed by alveolar macrophage 2 Site for hemineglect: right parietal | = Patient smells rubber. Mass located in left temporal lobe. Visual field defect: right upper quadrantanopia © Pineal gland: site for melatonin formation a44 SI Note: This material is copyrighted. All rights reserved. Edward Goljan, M.D. 2002 v4 94 944 Superficial cerebral veins: drain into superior sagital sinus Site odontoblasts develop from: * odontoblasts develop into dentin, « odontobl aspect of the developing tooth, * outer portion develops enamel from ameloblasts lasts are on inner HIGH YIELD NOTES BIOCHEMISTRY © Hep finsesavaea « type Land V have chylomicrons associated with them, © type Tis increase in LDL, # is remnant disease (dysbetal . Vand ope V ape (dysberalipoproteinemia), « increase in VLDL in type PCR mechanism: uses DNA polymerase to break down DNA into fragments Locations of biochemical processes in cell: « cytosol * glycolysis, * pentose phosphate shunt, « FA synthesis, * glycogen synthesis, * mitochondrial matrix * B-oxidation of FAs, « TCA cycle, inner mitochondrial membrane- oxidative phosphorylation, © both cytosol and mitochondria « gluconeogenesis, * urea cycle, * heme synthesis ‘Neurotransmitter from an essential amino acid: serotonin coming from tryptophan Brain energy during starvation: * ketone bodies, » uses glucose during fed and fasting state Gluconeogewic enrymes: * pyruvate carboxylase, * phosphoenolpyruvate carboxykinase, « fructose 1,6-bisphosphatase (rate limiting), * glucose 6-phosphatase (deficient in von Gierke's glycogenosis) RBC ribosomes: + lost after RBC leaves bone marrow, * persistence in peripheral blood produces basophilic stippling, * coarse basophilic suppling sign of Pb poisoning (ribonuclease denatured by Pb) B Thalassemia mechanisms: « most often a splicing defect for mild forms, « severe B-thal is due toa stop codon preventing 8-chain transcription Man with 2900 calorie diet with 30% of it representing fat, how many grams is fat: © fat has 9 cal/g, + 2900 x 0.30 = 870 calories is fat, « 870 + 9 =~97 grams Vas and Ke: © glucokinase high Km (low affinity for glucose) and high Vm (only reacts with glucose), « hexokinase low Km (high affinity for glucose, good for fasting’state) and low Vin (reacts with all hexose sugars) Essential fatty acids (FAs): * linoleic acid- + C18:206, * produces arachidonic acid, * not cardioprotective, + com oil/safflower oil, ¢ c-linolenic— = C18:303, * cardioprotective (lower triglycerides, inhibit platelet aggregation, produce anti-inflammatory prostaglandins, less damage to myocardial tissue in infarctions, * found in fish oils, canola oil (best oil) I cell disease: inability 10 phosphorylate the mannose residues of potential lysosomal enzymes located in Golgi apparatus, hence they cannot be taken up by the lysosomes to degrade complex substrates ber of glucoses necessary to build palmitic acid a 16 carbon compound: 4 glucoses, cach glucose rum produces 2 acetyl CoA, the latter containing 2 carbons cach - Insalin lack in DKA: decreased glycolysis, glycogenesis, farry acid synthesis, storage of fat in adi Compara Chart of the Well Fed State, Fasting State, and Starved State: Well fed state ~ Fasting state [Starved state Glycogenesis Increased fone ‘None Glycogenolysis Decreased: none in| Increased: early supply | None: glycogen used up the liver, some in _| of glucose derived from muscic liver not muscle $2 Note: This material is copyrighted. All rights reserved. Edward Goljan, M.D, 2002 ‘Wall fed state Fasting state Sin Gluconeogenesis None Increased: primary] Beteased jan cnan = ] source of glucose after | supply RBCs lysis Triacylglycerol synthesis | increased None Tee : in liver/adipose Lipolysis None Tnereased [increased Fate of glycerol ‘Synthesize more ‘Substrate for Substrate for riacylglycero! in liver | gluconeogenesis [eee | Brosidation of arty acids | None Increased Markedly increased: primary fuel for muscle Muscle catabolism None: inereased Increased: supply amino | Decreased: conserve | protein synthesis and | acids for ‘muscle for important body | t uptake of aming acids | gluconeogenesis functions i ‘Urea synthesivexcretion [Remains constant: | Increased: deamination | Decreased: less muscle | handles NH." load | of amino acids used for | breakdown of protein with | from protein gluconeogenesis less amino acids to degradation in gut by | increases urea synthesis | degrade 2 bacteria i ‘Ketone body syathesis | None Tncreased Markedly increased: by- | product ofacetyiCoA from increased B- | : oxidation of fatty acids | [Muscle use of glucose for | Primary Fuel Decreased ‘None: mainly uses fatty fuel . acids )Musele use of fatty acids | None Increased: primaty fuel {Markedly increased, for fuel primary fuel ! jMasele use of ketones | None ‘Some: alternative fuel | None: allows the brain to for fuel use ketones for fuel Brain use of glucose for [Remains constant | Remains constant Decreased: allows RBCs | | fuel to primarily use glucose for fuel [Brain use of Ketones for, [None None Increased: primary fust fuel i TRBC use of glucose for [Remains constant | Remains constant Remains constant 1 fuel ; = Acquired and Genetic Hyperlipoproteinemias: Lipid Disorders ‘Clinical Comments [Laboratory Findings —____ ‘Type Rare childhood disease. Tnoreased chylomicrons and TG with + Familia! lipoprotein ‘anormal cholesterol and LDL, «stan | | lipase deficiency ing chylomicron test: supranate but no | Apo ci pyaar Pathogenesis: cannot hydrolyze chylomicrons 53 Note: This material is copyrighted. All rights reserved. Edward Goljan, MLD. 2007 Lipid Disorders ‘Glinical Comments Laboratory Findings Type tl + AD disorder with premature | » Type Ila: increased ‘+ Familial hypercholester- | CAD. « Achilles tendon xanth- melt) and Eanes To. ‘olerma ‘omas pathognomonic, * Rx: type Ib: increased LDL, cholesterol, Pathogenesis: absent or | “statin” drugs and TG. " defective LDL receptors | « Acquired causes: diabetes, 2 hypothyroidism, obstructive i jaundice, progesterone in birth | ‘control pills. i Type * Increased CAD risk, » hyperun- |» Choleserol and TG equally levarsa. = Famitial cemia, + obesity, « diabetes + crease in chylorucron and IDL dteclipoprotenemia remnants, ¢ ultracentrifugation followed | remnant disease” lectrophoresis eran Pathogenesis: deficiency we “donates ™ ‘of apo E. Chylomicron and DL remmants are not metabolized in the liver. Type IV ‘+ AD disorder, « most common | « Marked increase in TG and slight + Familial byperlipoproteinemia, « TG increase in cholesterol, « standing hypertnglyceridemia begins increasing at puberty, | chylomicron test: turbid infranate, « Pathogenesis: decreased | increased incidence of CAD and | HDL decreased (inverse relanoaship caabolism or increased | perspheral vascular disease, « Rx: | with VLDL). | synthesis of VLDL. fibric acid derivatives, reduce carbohydrate and alcohol intake. + Acquired causes: alcobolism, diuretics, B-blockers, renal failure. Type ¥ ‘+ Particularly common in ‘+ Markedly increased TG with normal Most commonly a alcoholics and DKA,* hyper | LDL, « standing chylomicron test. familial chylomicronemia syndrome: supranste and infranate hypertriglyceridemia with | abdominal pain. pancreatitis, exacerbanng factors dyspnea (impaired oxygen Pathogenesis: exchange), hepatospleaomegaly combination of type Iand | (fatty change), papules on skin. type IV mechanisms. CAD = coronary artery disease, DKA = diabetic ketoacidosis, IDL = imcrmediare density ipopromm, LDL = low density lipoproteins, Rx = teament, TG - triacylglycerol, VLDL * very low density lipoproteins ‘Rate limiting reaction in cholesterol synthesis: « HMG Co reductase: inhibited by cholesterol and statin drags (competitive inhibition with mevalonic acid), = enzyme converts HMG CoA into mevalonic acid Rate limiting reaction in fatty acid syathesis: » acetyl CoA carboxylase, * enzyme converts acetyl CoA into malonyl Com ~ Rate limiting reaction in Boxidation of farry acids: « carnitine acyltransferase I, ¢ enzyme in ‘outer membrane of mitochonéria removes acy! group from farty acyl CoA and wansfers it 10 cammitine (acyleamitine), © mner membrane enzyme removes acyl group from acylcamitine and transfers it back to CoA to produce fatty acyl CoA Rate limiting reaction in glycolysis: « phosphofructokinase [, + enzyme converts fructose 6 phosphate into fructose 1,6-bisphosphate q q Rate limiting reaction in gluconeogenesis: © fructose 1,6-bisp! fructose 1,6-bisphosphate to fructose 6-phosphate tose |.6-bisphosphatase, * enzyme convens Rate limiting reaction in glycogen synthesis: « . glycosidic linkages between a glucose unit from ior eae nonredci or oad existing glycogen chain icing end of an Rate Umiting reaction in glycogenolysis: « liver eer (releases glucose I-phosphate) but stops working Apietiecia from seen aiaess Rate limiting reaction in pentose phosphate pathway: « ae Peay tece te wiasece B phenrtat Wed phngieglengets Bait Sehesclieelapiosgrume Rate limiting reaction in lipolysis: * hormone sensiti 3 ane miacylglycerol into farty acids and glycerol ves geal, Rate limiting reaction in urea cycle: carbamyl phosphate F enzymeeon NEL + ATP ino carbamyl phosphate mi phosphate syrihase I converts COs + Rate limiting reaction in pyrimidine metabolism: « carbamy! ph acai see Teens . Rate limiting resction in purine metabolism: + glutamine PRPP aminotransferase, + enzyme converts PRPP + glutamine into 5‘-phosphoriboysylamine Rate limiting reaction in heme synthesis: « ALA synthase, * enzyme converss succinyl CoA + glycine into 5-aminolevulinic acid Rate limiting reaction in ketone body synthesis: © HMG CoA syathase, * converts acetoacetyl CoA into HMG CoA Tncoupling agents: « render the inner mitochondrial membrane permeable and carry protons with then into’ the mitochondrial matrix (destroys the proton gradient): eg, dinitrophenol, pentachlorophenol (used 10 teat wood to prevent insect invasion), thermogeain (natal incoupling agent in brown fat of newborns that helps keep newborns intemal temperatures highet), « rate of chemical reactions increases to produce more NADH and NADPH since its being siphoned off into the mitochondrial matrix without the synthesis of ATP—> poteatial for typerthermia, «mitochondrial poisons (aleobol, slicytates) also reader the inner mitochondrial wot iorane permeable to protons, but they do not directly take protons with them into the matrix (not true uncoupling agents): | ATP synthesis ejent with pheochromocytoma: « decrease tyrosine (not an essential AA) in the diet, since itis the precursor for the catecholamines, « also decreased ascorbic acid which converts dopamine into norepinephrine Tesch Nyban: « SXR with absent HGPRT, + self-mutilation, » hyperuricemia, ¢ mental retardation Glucokinase and hexokinase: « glucokinase: only in liv, high Vm and high Km, nat inhibited by Slucose é-phospbate, « hexolanase: in all tissues, inhibited by glucose G-phosphate, low Vm and Tow Kim Branched chain amino acids and maple syrup urine disease: ¢ only muscle can metabolize teanched chain amino acids, © missing dehydrogenase enzyme Locations of glucose 6-phosphatase (glacomeogenic hormone): + liver, * kidney, + itesunal epithelium (lesser extent than others), + absent in voa Gierke’s disease qeaities and what they carry: + camitine- even chained fay acids, malate and glyoerel 5 ~ pkosphaté—NADE™ venedons of cholesterol: « vitamiri D synthesis in the skin (7-dchydrocholesterol), © steroid synthesis, © cell membranes, + synthesis of bile saltsacids ‘Acetyl CoA uses: # FA synthesis, » CH synthesis, « ketone body synthesis. ¢ synthesis of citrate along with oxaloacetic acid, + not a sut 35 re ) Ketone body synthesis: + FIMG CoA synthase is the ratelimiti converts HMG CoA into acetoacetic acid, while in cbileirsl atte ieee lyase | converts HMG CoA into mevalonic acid, « 23 ATP produced for fuel a Enzyme kinetics: * Vu. ropresents the maximum velocity of an sites are fully saturated, » K., (Michaelis-Menton constant) of an enzyme concentration at which the reaction velocity is one-half of Vmax (Vina) ie dae enzymes affinity for substrate, + increased Ka indicates decreased affinity of the ee | substrate, + decreased Ky indicates increased affinity of the enzyme for subsets Competitive inhibitors: * Vies is not changed since the reaction rate is unchanged whether the competitive inhibitor (e.g. ethylene glycol, methyl alcohol) is binding to the active site of the enzyme (e.g, alcohol dehydrogenase) or alcohol, + Ky increases (decrease in the affinity of the enzyme for alcohol), since the enzyme is also actively binding with ethylene glycalimethyl alcobel + infusing alcohol decreases the metabolism of ethylene glycol/methyl alcohol, hence the effect of a competitive inhibitor is reversed by increasing substrate, + methotrexate is also 2 compentive itor Reversible noncompetitive inhibitors: * Vea is decreased, however the Ke remains the same, « reversible noncompetitive inhibitors bind reversibly (non-covalent bonds) away from the active binding site of the enzyme and form either unreactive enzyme-inhibitor complexes or enzyme. substrate-inhibitor complexes, + affinity of the enzyme for substrate is unchanged (K,.), since the active binding site is unaltered and normally binds with substrate, # Vy is decreased since the inhibitor inactivates the enzyme, which automatically decreases the eflective covcentration of active enzyme, * increasing substrate concentration does not reverse the effect of the inhibitor, since the inhibitor is blocking enzyme activity away from the active binding site, » examples of reversible noncompetitive inhibitors include angiotensin converting enzyme inhibitors (substrate is angiotensin I); physostigmine, which is a cholinesterase inhibitor (substrate acetylcholine); and, allopurinol, an xanthine oxidase inhibitor (substrate xanthine), * irreversible inhibitors permanently inactivate enzymes by forming strong covalent bonds: examples include lead (irreversibly inhibits ferrochelatase and S-aminolevulinic acid dehydrase), aspirin (irreversibly — inhibits platelet cyclooxygenase), and organophosphates (irreversibly inhibit cholinesterase), enzyme Kinetics are the same as those for reversible noncompetitive inhibitors ATP count using palmitic acid as an example: (1) _ divide the number of carbons in the FA by 2 to arrive at the number of acetyl CoAs produced: | palmitic acid = 16 carbons 8 acetyl CoA. (2) cach acetyl CoA produces 12 ATP when converted to CO; and H,0 in the TCA cycle: 8 x 12 | =96 ATP (3) subtract 1 from the number of acetyl CoA produced to arrive at the number of NADH: and FADE) produced: 7 NADH +7 FADE; (4) each NADH produces 3 ATP: 7x3 =21 | (5) each FADH; produces 2 ATP: 7x2= 14 (©) tol ATP= 131 ; ' Urea cycle: « method of eliminating ammonia, « located in the hepatocyte, » chronic liver disease: low BUN, elevated ammonia Epinephrine given and only small branched chaios of glycogen found: debrancher enzyme | deficiency 4 Cholesterol synthesis: first few steps are similar to ketone body synthesis except HMG CoA Tyase is used instead of HMG CoA reductase Apolipoprotein 100 (liver) and 48 (intestine) | 56 944 a4 Tasulin: * key hormone of the fed state,» activates phosphatase i" Glucagon: » key hormone of the fasting statc, « eae pel ae ‘ohotpy Mannose 6 phosphate: « involved in transfer of dolichol (lipid) in the =e linked glycosides, involved in transfer of lysosomes from Golgi a acid—arginine Mechanism of ketoacidosis in DKA: increased ti i CoA, which is used by the liver to synthesize Sater Of any acid and prodveton of seen Promoter location: upstream location Energy source for protein synthesis: GIP Isvenzyme with 2 genes, 4 subunits: LDH isoenzymes; 5 isotypes: LLLL, LLLH, LLHH, LHEDH, Second messengers: atrial natriuretic peptide: eGMP, insulin: tyrosine kinase, nicotinic: ion channels Best method of detecting relatedness of a new bacteria: restriction fragment Iength polymorphism Biochemistry of hepatic encephalopathy: * increase in aromatic amino acids phenylalanine, tyrosine, tryptophan (mnemonic— PTT) Icads to increased synthesis of false neurotransmutters (GABA, octopamine) in hepatic encephalopathy, « branched chain amino acids inhibit synthesis of false neurotransmitters- reason why they are given for Rx Energy in cardiac muscle: B-oxidation of fatty acids Lipid facts: * fimctions of HDL- = reservoir for apolipoproteins in the blood, * transport of esterified cholesterol to the liver, * transfers cholesterol esters to VEDL in exchange for triglyceride using cholesterol ester transport protein, « taken up by scavenger receptors in the liver, * in patients with diabetic ketoacidasis you would expect... activation of hormone sensitive lipase in the adipose, conversion of glycerol 3-phosphate into dihydroxyacetone phosphate, «in the fasting state, you would expect~ loss of inhibition of camitine acyltransferase by malonyl-CoA, + in the fed state, you would expect- * activation of citrate lyase in the cytosol, * inactivation of hormone sensitive lipase in adipose, + increased production of palmitic acid in the cytosol Findings in PKU: « AR discase with deficiency of phenvislanine hydroxylase, * newborn must be exposed to phenylalanine (PHY) in the diet before it is increased, © mousy odor, « projectile vomiting simulating congenital pyloric stenosis, « tyrosine missing, hence it must be supplied in the diet, + can diagnose by amniocentesis and finding the abnormal gene, « eliminate phenylalanine from dict Nuzasweet is aspartate and phenylalanine, s0 cannot use it, « woman with PKU who is pregnant must be on a PHY free diet— affected fetus will develop permanent CNS damage inf utero owing to exposure of PHY . Disorders of galactose metabolism: * galactose derives from lactose metabolism lactose + lactase glucose + galactose, » galactose metabolism in sequence is as follows- * galactose + galactokinase > galactose 1-PO,, + galactose 1-PO, + GALT (galactose-I-phosphate uridyl wansferase) + UDP-glucose glucose 1-PO, + UDP-galactose, + glucose 1-PO. + ST

You might also like